exam 3 med surg review 1

Pataasin ang iyong marka sa homework at exams ngayon gamit ang Quizwiz!

Which client does the medical unit charge nurse assign to a licensed practical nurse (LPN)/licensed vocational nurse (LVN)? a. A client with chronic microcytic anemia associated with alcohol use b. A client scheduled for a bone marrow biopsy with conscious sedation c. A client with a history of a splenectomy and a temperature of 100.9°F (38.3°C) d. A client with atrial fibrillation and an international normalized ratio of 6.6

A

A nurse prepares a client with acute renal insufficiency for a cardiac catheterization. The provider prescribes 0.9% normal saline to infuse at 125 mL/hr for renal protection. The nurse obtains gravity tubing with a drip rate of 15 drops/mL. At what rate (drops/min) should the nurse infuse the fluids? (Record your answer using a whole number, and rounding to the nearest drop.) _____ drops/min

ANS: 31 drops/min DIF: Applying/Application REF: 641 KEY: Medication administration MSC: Integrated Process: Nursing Process: Implementation NOT: Client Needs Category: Physiological Integrity: Pharmacological and Parenteral Therapies

A nurse is caring for a client with a nonhealing arterial lower leg ulcer. What action by the nurse is best? a. Consult with the Wound Ostomy Care Nurse. b. Give pain medication prior to dressing changes. c. Maintain sterile technique for dressing changes. d. Prepare the client for eventual amputation.

ANS: A A nonhealing wound needs the expertise of the Wound Ostomy Care Nurse (or Wound Ostomy Continence Nurse). Premedicating prior to painful procedures and maintaining sterile technique are helpful, but if the wound is not healing, more needs to be done. The client may need an amputation, but other options need to be tried first. DIF: Applying/Application REF: 734 KEY: Peripheral vascular disease| consultation| wound care MSC: Integrated Process: Communication and Documentation NOT: Client Needs Category: Safe and Effective Care Environment: Management of Care

A nurse is interested in providing community education and screening on hypertension. In order to reach a priority population, to what target audience should the nurse provide this service? a. African-American churches b. Asian-American groceries c. High school sports camps d. Women's health clinics

ANS: A African Americans in the United States have one of the highest rates of hypertension in the world. The nurse has the potential to reach this priority population by providing services at African-American churches. Although hypertension education and screening are important for all groups, African Americans are the priority population for this intervention. DIF: Remembering/Knowledge REF: 711 KEY: Hypertension| primary prevention| secondary prevention| cultural awareness MSC: Integrated Process: Nursing Process: Analysis NOT: Client Needs Category: Health Promotion and Maintenance

A nurse is teaching a larger female client about alcohol intake and how it affects hypertension. The client asks if drinking two beers a night is an acceptable intake. What answer by the nurse is best? a. "No, women should only have one beer a day as a general rule." b. "No, you should not drink any alcohol with hypertension." c. "Yes, since you are larger, you can have more alcohol." d. "Yes, two beers per day is an acceptable amount of alcohol."

ANS: A Alcohol intake should be limited to two drinks a day for men and one drink a day for women. A "drink" is classified as one beer, 1.5 ounces of hard liquor, or 5 ounces of wine. Limited alcohol intake is acceptable with hypertension. The woman's size does not matter. DIF: Understanding/Comprehension REF: 712 KEY: Hypertension| lifestyle choices| patient education MSC: Integrated Process: Teaching/Learning NOT: Client Needs Category: Health Promotion and Maintenance

A nurse is caring for a client who is intubated and has an intra-aortic balloon pump. The client is restless and agitated. What action should the nurse perform first for comfort? a. Allow family members to remain at the bedside. b. Ask the family if the client would like a fan in the room. c. Keep the television tuned to the client's favorite channel. d. Speak loudly to the client in case of hearing problems

ANS: A Allowing the family to remain at the bedside can help calm the client with familiar voices (and faces if the client wakes up). A fan might be helpful but may also spread germs through air movement. The TV should not be kept on all the time to allow for rest. Speaking loudly may agitate the client more. DIF: Applying/Application REF: 780 KEY: Intra-aortic balloon pump| nonpharmacologic comfort measures MSC: Integrated Process: Nursing Process: Implementation NOT: Client Needs Category: Physiological Integrity: Basic Care and Comfort

A nurse is assessing a client with peripheral artery disease (PAD). The client states walking five blocks is possible without pain. What question asked next by the nurse will give the best information? a. "Could you walk further than that a few months ago?" b. "Do you walk mostly uphill, downhill, or on flat surfaces?" c. "Have you ever considered swimming instead of walking?" d. "How much pain medication do you take each day?"

ANS: A As PAD progresses, it takes less oxygen demand to cause pain. Needing to cut down on activity to be pain free indicates the client's disease is worsening. The other questions are useful, but not as important. DIF: Applying/Application REF: 719 KEY: Pain| exercise| activity| peripheral vascular disease| pain assessment MSC: Integrated Process: Nursing Process: Assessment NOT: Client Needs Category: Physiological Integrity: Physiological Adaptation

A nurse assesses a client with tachycardia. Which clinical manifestation requires immediate intervention by the nurse? a. Mid-sternal chest pain b. Increased urine output c. Mild orthostatic hypotension d. P wave touching the T wave

ANS: A Chest pain, possibly angina, indicates that tachycardia may be increasing the client's myocardial workload and oxygen demand to such an extent that normal oxygen delivery cannot keep pace. This results in myocardial hypoxia and pain. Increased urinary output and mild orthostatic hypotension are not life-threatening conditions and therefore do not require immediate intervention. The P wave touching the T wave indicates significant tachycardia and should be assessed to determine the underlying rhythm and cause; this is an important assessment but is not as critical as chest pain, which indicates cardiac cell death. DIF: Applying/Application REF: 663 KEY: Cardiac electrical conduction MSC: Integrated Process: Nursing Process: Assessment NOT: Client Needs Category: Safe and Effective Care Environment: Management of Care

A nursing student is caring for a client who had a myocardial infarction. The student is confused because the client states nothing is wrong and yet listens attentively while the student provides education on lifestyle changes and healthy menu choices. What response by the faculty member is best? a. "Continue to educate the client on possible healthy changes." b. "Emphasize complications that can occur with noncompliance." c. "Tell the client that denial is normal and will soon go away." d. "You need to make sure the client understands this illness."

ANS: A Clients are often in denial after a coronary event. The client who seems to be in denial but is compliant with treatment may be using a healthy form of coping that allows time to process the event and start to use problem-focused coping. The student should not discourage this type of denial and coping, but rather continue providing education in a positive manner. Emphasizing complications may make the client defensive and more anxious. Telling the client that denial is normal is placing too much attention on the process. Forcing the client to verbalize understanding of the illness is also potentially threatening to the client. DIF: Understanding/Comprehension REF: 769 KEY: Coronary artery disease| psychosocial response| coping| therapeutic communication MSC: Integrated Process: Communication and Documentation NOT: Client Needs Category: Psychosocial Integrity

A client has peripheral arterial disease (PAD). What statement by the client indicates misunderstanding about self-management activities? a. "I can use a heating pad on my legs if it's set on low." b. "I should not cross my legs when sitting or lying down." c. "I will go out and buy some warm, heavy socks to wear." d. "It's going to be really hard but I will stop smoking."

ANS: A Clients with PAD should never use heating pads as skin sensitivity is diminished and burns can result. The other statements show good understanding of self-management. DIF: Evaluating/Synthesis REF: 722 KEY: Peripheral arterial disease| patient education| patient safety MSC: Integrated Process: Nursing Process: Evaluation NOT: Client Needs Category: Health Promotion and Maintenance

A client had an inferior wall myocardial infarction (MI). The nurse notes the client's cardiac rhythm as shown below: What action by the nurse is most important? a. Assess the client's blood pressure and level of consciousness. b. Call the health care provider or the Rapid Response Team. c. Obtain a permit for an emergency temporary pacemaker insertion. d. Prepare to administer antidysrhythmic medication.

ANS: A Clients with an inferior wall MI often have bradycardia and blocks that lead to decreased perfusion, as seen in this ECG strip showing sinus bradycardia. The nurse should first assess the client's hemodynamic status, including vital signs and level of consciousness. The client may or may not need the Rapid Response Team, a temporary pacemaker, or medication; there is no indication of this in the question. DIF: Analyzing/Analysis REF: 769 KEY: Coronary artery disease| dysrhythmias| nursing assessment| hemodynamic status MSC: Integrated Process: Nursing Process: Implementation NOT: Client Needs Category: Physiological Integrity: Physiological Adaptation

A client is in the clinic a month after having a myocardial infarction. The client reports sleeping well since moving into the guest bedroom. What response by the nurse is best? a. "Do you have any concerns about sexuality?" b. "I'm glad to hear you are sleeping well now." c. "Sleep near your spouse in case of emergency." d. "Why would you move into the guest room?"

ANS: A Concerns about resuming sexual activity are common after cardiac events. The nurse should gently inquire if this is the issue. While it is good that the client is sleeping well, the nurse should investigate the reason for the move. The other two responses are likely to cause the client to be defensive. DIF: Applying/Application REF: 781 KEY: Coronary artery disease| sexuality| anxiety| therapeutic communication MSC: Integrated Process: Caring NOT: Client Needs Category: Psychosocial Integrity

A client is on a dopamine infusion via a peripheral line. What action by the nurse takes priority for safety? a. Assess the IV site hourly. b. Monitor the pedal pulses. c. Monitor the client's vital signs. d. Obtain consent for a central line.

ANS: A Dopamine should be infused through a central line to prevent extravasation and necrosis of tissue. If it needs to be run peripherally, the nurse assesses the site hourly for problems. When the client is getting the central line, ensuring informed consent is on the chart is a priority. But at this point, the client has only a peripheral line, so caution must be taken to preserve the integrity of the client's integumentary system. Monitoring pedal pulses and vital signs give indications as to how well the drug is working. DIF: Applying/Application REF: 773 KEY: Inotropic agents| adverse effects| medication safety MSC: Integrated Process: Nursing Process: Assessment NOT: Client Needs Category: Physiological Integrity: Pharmacological and Parenteral Therapies

An emergency room nurse obtains the health history of a client. Which statement by the client should alert the nurse to the occurrence of heart failure? a. "I get short of breath when I climb stairs." b. "I see halos floating around my head." c. "I have trouble remembering things." d. "I have lost weight over the past month."

ANS: A Dyspnea on exertion is an early manifestation of heart failure and is associated with an activity such as stair climbing. The other findings are not specific to early occurrence of heart failure. DIF: Applying/Application REF: 635 KEY: Health screening| heart failure MSC: Integrated Process: Nursing Process: Assessment NOT: Client Needs Category: Health Promotion and Maintenance

A client had a femoropopliteal bypass graft with a synthetic graft. What action by the nurse is most important to prevent wound infection? a. Appropriate hand hygiene before giving care b. Assessing the client's temperature every 4 hours c. Clean technique when changing dressings d. Monitoring the client's daily white blood cell count

ANS: A Hand hygiene is the best way to prevent infections in hospitalized clients. Dressing changes should be done with sterile technique. Assessing vital signs and white blood cell count will not prevent infection. DIF: Applying/Application REF: 724 KEY: Infection control| hand hygiene| wound infection MSC: Integrated Process: Nursing Process: Implementation NOT: Client Needs Category: Safe and Effective Care Environment: Safety and Infection Control

The nurse is caring for four hypertensive clients. Which drug-laboratory value combination should the nurse report immediately to the health care provider? a. Furosemide (Lasix)/potassium: 2.1 mEq/L b. Hydrochlorothiazide (Hydrodiuril)/potassium: 4.2 mEq/L c. Spironolactone (Aldactone)/potassium: 5.1 mEq/L d. Torsemide (Demadex)/sodium: 142 mEq/L

ANS: A Lasix is a loop diuretic and can cause hypokalemia. A potassium level of 2.1 mEq/L is quite low and should be reported immediately. Spironolactone is a potassium-sparing diuretic that can cause hyperkalemia. A potassium level of 5.1 mEq/L is on the high side, but it is not as critical as the low potassium with furosemide. The other two laboratory values are normal. DIF: Applying/Application REF: 714 KEY: Hypertension| antihypertensive medications| laboratory values MSC: Integrated Process: Nursing Process: Analysis NOT: Client Needs Category: Physiological Integrity: Reduction of Risk Potential

A client is taking warfarin (Coumadin) and asks the nurse if taking St. John's wort is acceptable. What response by the nurse is best? a. "No, it may interfere with the warfarin." b. "There isn't any information about that." c. "Why would you want to take that?" d. "Yes, it is a good supplement for you."

ANS: A Many foods and drugs interfere with warfarin, St. John's wort being one of them. The nurse should advise the client against taking it. The other answers are not accurate. DIF: Understanding/Comprehension REF: 733 KEY: Anticoagulants| herbs and supplements| medication-food interactions| patient education MSC: Integrated Process: Teaching/Learning NOT: Client Needs Category: Physiological Integrity: Pharmacological and Parenteral Therapies

A nurse assesses a client who is recovering from a myocardial infarction. The client's pulmonary artery pressure reading is 25/12 mm Hg. Which action should the nurse take first? a. Compare the results with previous pulmonary artery pressure readings. b. Increase the intravenous fluid rate because these readings are low. c. Immediately notify the health care provider of the elevated pressures. d. Document the finding in the client's chart as the only action.

ANS: A Normal pulmonary artery pressures range from 15 to 26 mm Hg for systolic and from 5 to 15 mm Hg for diastolic. Although this client's readings are within normal limits, the nurse needs to assess any trends that may indicate a need for medical treatment to prevent complications. There is no need to increase intravenous fluids or notify the provider. DIF: Applying/Application REF: 638 KEY: Coronary perfusion| assessment/diagnostic examination| vascular perfusion MSC: Integrated Process: Nursing Process: Implementation NOT: Client Needs Category: Safe and Effective Care Environment: Management of Care

An older adult is on cardiac monitoring after a myocardial infarction. The client shows frequent dysrhythmias. What action by the nurse is most appropriate? a. Assess for any hemodynamic effects of the rhythm. b. Prepare to administer antidysrhythmic medication. c. Notify the provider or call the Rapid Response Team. d. Turn the alarms off on the cardiac monitor.

ANS: A Older clients may have dysrhythmias due to age-related changes in the cardiac conduction system. They may have no significant hemodynamic effects from these changes. The nurse should first assess for the effects of the dysrhythmia before proceeding further. The alarms on a cardiac monitor should never be shut off. The other two actions may or may not be needed. DIF: Applying/Application REF: 769 KEY: Coronary artery disease| older adult| pathophysiology| nursing assessment MSC: Integrated Process: Nursing Process: Assessment NOT: Client Needs Category: Health Promotion and Maintenance

A nurse teaches a client who experiences occasional premature atrial contractions (PACs) accompanied by palpitations that resolve spontaneously without treatment. Which statement should the nurse include in this client's teaching? a. "Minimize or abstain from caffeine." b. "Lie on your side until the attack subsides." c. "Use your oxygen when you experience PACs." d. "Take amiodarone (Cordarone) daily to prevent PACs."

ANS: A PACs usually have no hemodynamic consequences. For a client experiencing infrequent PACs, the nurse should explore possible lifestyle causes, such as excessive caffeine intake and stress. Lying on the side will not prevent or resolve PACs. Oxygen is not necessary. Although medications may be needed to control symptomatic dysrhythmias, for infrequent PACs, the client first should try lifestyle changes to control them. DIF: Applying/Application REF: 663 KEY: Patient education| cardiac electrical conduction MSC: Integrated Process: Teaching/Learning NOT: Client Needs Category: Health Promotion and Maintenance

A nurse assesses a client who has aortic regurgitation. In which location in the illustration shown below should the nurse auscultate to best hear a cardiac murmur related to aortic regurgitation? a. Location A b. Location B c. Location C d. Location D

ANS: A The aortic valve is auscultated in the second intercostal space just to the right of the sternum. DIF: Applying/Application REF: 639 KEY: Assessment/diagnostic examination MSC: Integrated Process: Nursing Process: Assessment NOT: Client Needs Category: Health Promotion and Maintenance

A client has been diagnosed with a deep vein thrombosis and is to be discharged on warfarin (Coumadin). The client is adamant about refusing the drug because "it's dangerous." What action by the nurse is best? a. Assess the reason behind the client's fear. b. Remind the client about laboratory monitoring. c. Tell the client drugs are safer today than before. d. Warn the client about consequences of noncompliance.

ANS: A The first step is to assess the reason behind the client's fear, which may be related to the experience of someone the client knows who took warfarin. If the nurse cannot address the specific rationale, teaching will likely be unsuccessful. Laboratory monitoring once every few weeks may not make the client perceive the drug to be safe. General statements like "drugs are safer today" do not address the root cause of the problem. Warning the client about possible consequences of not taking the drug is not therapeutic and is likely to lead to an adversarial relationship. DIF: Applying/Application REF: 709 KEY: Psychosocial response| anticoagulants| therapeutic communication| patient-centered care MSC: Integrated Process: Nursing Process: Assessment NOT: Client Needs Category: Psychosocial Integrity

A nurse prepares to discharge a client with cardiac dysrhythmia who is prescribed home health care services. Which priority information should be communicated to the home health nurse upon discharge? a. Medication reconciliation b. Immunization history c. Religious beliefs d. Nutrition preferences

ANS: A The home health nurse needs to know current medications the client is taking to ensure assessment, evaluation, and further education related to these medications. The other information will not assist the nurse to develop a plan of care for the client. DIF: Applying/Application REF: 673 KEY: Hand-off communication MSC: Integrated Process: Communication and Documentation NOT: Client Needs Category: Safe and Effective Care Environment: Management of Care

A nurse is working with a client who takes atorvastatin (Lipitor). The client's recent laboratory results include a blood urea nitrogen (BUN) of 33 mg/dL and creatinine of 2.8 mg/dL. What action by the nurse is best? a. Ask if the client eats grapefruit. b. Assess the client for dehydration. c. Facilitate admission to the hospital. d. Obtain a random urinalysis.

ANS: A There is a drug-food interaction between statins and grapefruit that can lead to acute kidney failure. This client has elevated renal laboratory results, indicating some degree of kidney involvement. The nurse should assess if the client eats grapefruit or drinks grapefruit juice. Dehydration can cause the BUN to be elevated, but the elevation in creatinine is more specific for a kidney injury. The client does not necessarily need to be admitted. A urinalysis may or may not be ordered. DIF: Applying/Application REF: 709 KEY: Laboratory values| statins| nursing assessment| medication-food interaction MSC: Integrated Process: Nursing Process: Assessment NOT: Client Needs Category: Physiological Integrity: Pharmacological and Parenteral Therapies

A nurse supervises an unlicensed assistive personnel (UAP) applying electrocardiographic monitoring. Which statement should the nurse provide to the UAP related to this procedure? a. "Clean the skin and clip hairs if needed." b. "Add gel to the electrodes prior to applying them." c. "Place the electrodes on the posterior chest." d. "Turn off oxygen prior to monitoring the client."

ANS: A To ensure the best signal transmission, the skin should be clean and hairs clipped. Electrodes should be placed on the anterior chest, and no additional gel is needed. Oxygen has no impact on electrocardiographic monitoring. DIF: Remembering/Knowledge REF: 652 KEY: Assessment/diagnostic examination| interdisciplinary team| unlicensed assistive personnel (UAP) MSC: Integrated Process: Communication and Documentation NOT: Client Needs Category: Safe and Effective Care Environment: Management of Care

A nurse cares for a client who is on a cardiac monitor. The monitor displayed the rhythm shown below: Which action should the nurse take first? a. Assess airway, breathing, and level of consciousness. b. Administer an amiodarone bolus followed by a drip. c. Cardiovert the client with a biphasic defibrillator. d. Begin cardiopulmonary resuscitation (CPR).

ANS: A Ventricular tachycardia occurs with repetitive firing of an irritable ventricular ectopic focus, usually at a rate of 140 to 180 beats/min or more. Ventricular tachycardia is a lethal dysrhythmia. The nurse should first assess if the client is alert and breathing. Then the nurse should call a Code Blue and begin CPR. If this client is pulseless, the treatment of choice is defibrillation. Amiodarone is the antidysrhythmic of choice, but it is not the first action. DIF: Applying/Application REF: 670 KEY: Cardiac electrical conduction| medical emergency MSC: Integrated Process: Nursing Process: Implementation NOT: Client Needs Category: Physiological Integrity: Physiological Adaptation

A nurse assesses a client who had a myocardial infarction and is hypotensive. Which additional assessment finding should the nurse expect? a. Heart rate of 120 beats/min b. Cool, clammy skin c. Oxygen saturation of 90% d. Respiratory rate of 8 breaths/min

ANS: A When a client experiences hypotension, baroreceptors in the aortic arch sense a pressure decrease in the vessels. The parasympathetic system responds by lessening the inhibitory effect on the sinoatrial node. This results in an increase in heart rate and respiratory rate. This tachycardia is an early response and is seen even when blood pressure is not critically low. An increased heart rate and respiratory rate will compensate for the low blood pressure and maintain oxygen saturations and perfusion. The client may not be able to compensate for long, and decreased oxygenation and cool, clammy skin will occur later. DIF: Applying/Application REF: 638 KEY: Coronary perfusion| hemodynamics MSC: Integrated Process: Nursing Process: Assessment NOT: Client Needs Category: Physiological Integrity: Physiological Adaptation

A nurse is caring for a client with a history of renal insufficiency who is scheduled for a cardiac catheterization. Which actions should the nurse take prior to the catheterization? (Select all that apply.) a. Assess for allergies to iodine. b. Administer intravenous fluids. c. Assess blood urea nitrogen (BUN) and creatinine results. d. Insert a Foley catheter. e. Administer a prophylactic antibiotic. f. Insert a central venous catheter.

ANS: A, B, C If the client has kidney disease (as indicated by BUN and creatinine results), fluids and Mucomyst may be given 12 to 24 hours before the procedure for renal protection. The client should be assessed for allergies to iodine, including shellfish; the contrast medium used during the catheterization contains iodine. A Foley catheter and central venous catheter are not required for the procedure and would only increase the client's risk for infection. Prophylactic antibiotics are not administered prior to a cardiac catheterization. DIF: Applying/Application REF: 643 KEY: Assessment/diagnostic examination MSC: Integrated Process: Nursing Process: Implementation NOT: Client Needs Category: Physiological Integrity: Reduction of Risk Potential

A nurse is caring for a client with a nonhealing arterial ulcer. The physician has informed the client about possibly needing to amputate the client's leg. The client is crying and upset. What actions by the nurse are best? (Select all that apply.) a. Ask the client to describe his or her current emotions. b. Assess the client for support systems and family. c. Offer to stay with the client if he or she desires. d. Relate how smoking contributed to this situation. e. Tell the client that many people have amputations.

ANS: A, B, C When a client is upset, the nurse should offer self by remaining with the client if desired. Other helpful measures include determining what and whom the client has for support systems and asking the client to describe what he or she is feeling. Telling the client how smoking has led to this situation will only upset the client further and will damage the therapeutic relationship. Telling the client that many people have amputations belittles the client's feelings. DIF: Applying/Application REF: 725 KEY: Psychosocial response| coping| support| therapeutic communication MSC: Integrated Process: Caring NOT: Client Needs Category: Psychosocial Integrity

A nursing student planning to teach clients about risk factors for coronary artery disease (CAD) would include which topics? (Select all that apply.) a. Advanced age b. Diabetes c. Ethnic background d. Medication use e. Smoking

ANS: A, B, C, E Age, diabetes, ethnic background, and smoking are all risk factors for developing CAD; medication use is not. DIF: Remembering/Knowledge REF: 760 KEY: Coronary artery disease| pathophysiology| patient education MSC: Integrated Process: Teaching/Learning NOT: Client Needs Category: Physiological Integrity: Physiological Adaptation

A nurse is teaching a client with premature ectopic beats. Which education should the nurse include in this client's teaching? (Select all that apply.) a. Smoking cessation b. Stress reduction and management c. Avoiding vagal stimulation d. Adverse effects of medications e. Foods high in potassium

ANS: A, B, D A client who has premature beats or ectopic rhythms should be taught to stop smoking, manage stress, take medications as prescribed, and report adverse effects of medications. Clients with premature beats are not at risk for vasovagal attacks or potassium imbalances. DIF: Remembering/Knowledge REF: 673 KEY: Patient education MSC: Integrated Process: Nursing Process: Implementation NOT: Client Needs Category: Health Promotion and Maintenance

A nurse is caring for a client on IV infusion of heparin. What actions does this nurse include in the client's plan of care? (Select all that apply.) a. Assess the client for bleeding. b. Monitor the daily activated partial thromboplastin time (aPTT) results. c. Stop the IV for aPTT above baseline. d. Use an IV pump for the infusion. e. Weigh the client daily on the same scale.

ANS: A, B, D Assessing for bleeding, monitoring aPTT, and using an IV pump for the infusion are all important safety measures for heparin to prevent injury from bleeding. The aPTT needs to be 1.5 to 2 times normal in order to demonstrate that the heparin is therapeutic. Weighing the client is not related. DIF: Applying/Application REF: 731 KEY: Anticoagulants| patient safety| injury prevention MSC: Integrated Process: Nursing Process: Intervention NOT: Client Needs Category: Safe and Effective Care Environment: Safety and Infection Control

A client has been bedridden for several days after major abdominal surgery. What action does the nurse delegate to the unlicensed assistive personnel (UAP) for deep vein thrombosis (DVT) prevention? (Select all that apply.) a. Apply compression stockings. b. Assist with ambulation. c. Encourage coughing and deep breathing. d. Offer fluids frequently. e. Teach leg exercises.

ANS: A, B, D The UAP can apply compression stockings, assist with ambulation, and offer fluids frequently to help prevent DVT. The UAP can also encourage the client to do pulmonary exercises, but these do not decrease the risk of DVT. Teaching is a nursing function. DIF: Applying/Application REF: 731 KEY: Deep vein thrombosis| delegation| unlicensed assistive personnel (UAP) MSC: Integrated Process: Communication and Documentation NOT: Client Needs Category: Safe and Effective Care Environment: Management of Care

A nursing student studying acute coronary syndromes learns that the pain of a myocardial infarction (MI) differs from stable angina in what ways? (Select all that apply.) a. Accompanied by shortness of breath b. Feelings of fear or anxiety c. Lasts less than 15 minutes d. No relief from taking nitroglycerin e. Pain occurs without known cause

ANS: A, B, D, E The pain from an MI is often accompanied by shortness of breath and fear or anxiety. It lasts longer than 15 minutes and is not relieved by nitroglycerin. It occurs without a known cause such as exertion. DIF: Remembering/Knowledge REF: 762 KEY: Coronary artery disease| pathophysiology MSC: Integrated Process: Teaching/Learning NOT: Client Needs Category: Physiological Integrity: Physiological Adaptation

A nurse teaches a client with a new permanent pacemaker. Which instructions should the nurse include in this client's teaching? (Select all that apply.) a. "Until your incision is healed, do not submerge your pacemaker. Only take showers." b. "Report any pulse rates lower than your pacemaker settings." c. "If you feel weak, apply pressure over your generator." d. "Have your pacemaker turned off before having magnetic resonance imaging (MRI)." e. "Do not lift your left arm above the level of your shoulder for 8 weeks."

ANS: A, B, E The client should not submerge in water until the site has healed; after the incision is healed, the client may take showers or baths without concern for the pacemaker. The client should be instructed to report changes in heart rate or rhythm, such as rates lower than the pacemaker setting or greater than 100 beats/min. The client should be advised of restrictions on physical activity for 8 weeks to allow the pacemaker to settle in place. The client should never apply pressure over the generator and should avoid tight clothing. The client should never have MRI because, whether turned on or off, the pacemaker contains metal. The client should be advised to inform all health care providers that he or she has a pacemaker. DIF: Applying/Application REF: 675 KEY: Cardiac electrical conduction| patient education MSC: Integrated Process: Teaching/Learning NOT: Client Needs Category: Health Promotion and Maintenance

A client is being discharged on warfarin (Coumadin) therapy. What discharge instructions is the nurse required to provide? (Select all that apply.) a. Dietary restrictions b. Driving restrictions c. Follow-up laboratory monitoring d. Possible drug-drug interactions e. Reason to take medication

ANS: A, C, D, E The Joint Commission's Core Measures state that clients being discharged on warfarin need instruction on follow-up monitoring, dietary restrictions, drug-drug interactions, and reason for compliance. Driving is typically not restricted. DIF: Remembering/Knowledge REF: 733 KEY: Patient safety| Core Measures| anticoagulants| patient education MSC: Integrated Process: Teaching/Learning NOT: Client Needs Category: Safe and Effective Care Environment: Management of Care

A nurse reviews a client's laboratory results. Which findings should alert the nurse to the possibility of atherosclerosis? (Select all that apply.) a. Total cholesterol: 280 mg/dL b. High-density lipoprotein cholesterol: 50 mg/dL c. Triglycerides: 200 mg/dL d. Serum albumin: 4 g/dL e. Low-density lipoprotein cholesterol: 160 mg/dL

ANS: A, C, E A lipid panel is often used to screen for cardiovascular risk. Total cholesterol, triglycerides, and low-density lipoprotein cholesterol levels are all high, indicating higher risk for cardiovascular disease. High-density lipoprotein cholesterol is within the normal range for both males and females. Serum albumin is not assessed for atherosclerosis. DIF: Applying/Application REF: 636 KEY: Assessment/diagnostic examination| health screening MSC: Integrated Process: Nursing Process: Assessment NOT: Client Needs Category: Physiological Integrity: Reduction of Risk Potential

A nurse cares for a client who is recovering from a right-sided heart catheterization. For which complications of this procedure should the nurse assess? (Select all that apply.) a. Thrombophlebitis b. Stroke c. Pulmonary embolism d. Myocardial infarction e. Cardiac tamponade

ANS: A, C, E Complications from a right-sided heart catheterization include thrombophlebitis, pulmonary embolism, and vagal response. Cardiac tamponade is a risk of both right- and left-sided heart catheterizations. Stroke and myocardial infarction are complications of left-sided heart catheterizations. DIF: Remembering/Knowledge REF: 643 KEY: Assessment/diagnostic examination MSC: Integrated Process: Nursing Process: Assessment NOT: Client Needs Category: Physiological Integrity: Reduction of Risk Potential

A nurse is caring for a client who had coronary artery bypass grafting yesterday. What actions does the nurse delegate to the unlicensed assistive personnel (UAP)? (Select all that apply.) a. Assist the client to the chair for meals and to the bathroom. b. Encourage the client to use the spirometer every 4 hours. c. Ensure the client wears TED hose or sequential compression devices. d. Have the client rate pain on a 0-to-10 scale and report to the nurse. e. Take and record a full set of vital signs per hospital protocol.

ANS: A, C, E The nurse can delegate assisting the client to get up in the chair or ambulate to the bathroom, applying TEDs or sequential compression devices, and taking/recording vital signs. The spirometer should be used every hour the day after surgery. Assessing pain using a 0-to-10 scale is a nursing assessment, although if the client reports pain, the UAP should inform the nurse so a more detailed assessment is done. DIF: Applying/Application REF: 768 KEY: Coronary artery disease| coronary artery bypass graft| delegation| activity| unlicensed assistive personnel (UAP) MSC: Integrated Process: Communication and Documentation NOT: Client Needs Category: Safe and Effective Care Environment: Management of Care

The nurse working in the emergency department knows that which factors are commonly related to aneurysm formation? (Select all that apply.) a. Atherosclerosis b. Down syndrome c. Frequent heartburn d. History of hypertension e. History of smoking

ANS: A, D, E Atherosclerosis, hypertension, hyperlipidemia, and smoking are the most common related factors. Down syndrome and heartburn have no relation to aneurysm formation. DIF: Remembering/Knowledge REF: 726 KEY: Peripheral vascular disorders| pathophysiology MSC: Integrated Process: Teaching/Learning NOT: Client Needs Category: Physiological Integrity: Physiological Adaptation

A nurse cares for a client with congestive heart failure who has a regular cardiac rhythm of 128 beats/min. For which physiologic alterations should the nurse assess? (Select all that apply.) a. Decrease in cardiac output b. Increase in cardiac output c. Decrease in blood pressure d. Increase in blood pressure e. Decrease in urine output f. Increase in urine output

ANS: A, D, E Elevated heart rates in a healthy client initially cause blood pressure and cardiac output to increase. However, in a client who has congestive heart failure or a client with long-term tachycardia, ventricular filling time, cardiac output, and blood pressure eventually decrease. As cardiac output and blood pressure decrease, urine output will fall. DIF: Applying/Application REF: 657 KEY: Cardiac electrical conduction| heart failure MSC: Integrated Process: Nursing Process: Assessment NOT: Client Needs Category: Physiological Integrity: Physiological Adaptation

A nurse is caring for a client with a deep vein thrombosis (DVT). What nursing assessment indicates a priority outcome has been met? a. Ambulates with assistance b. Oxygen saturation of 98% c. Pain of 2/10 after medication d. Verbalizing risk factors

ANS: B A critical complication of DVT is pulmonary embolism. A normal oxygen saturation indicates that this has not occurred. The other assessments are also positive, but not the priority. DIF: Analyzing/Analysis REF: 729 KEY: Pulmonary embolism| deep vein thrombosis| respiratory assessment| thromboembolic event MSC: Integrated Process: Nursing Process: Assessment NOT: Client Needs Category: Physiological Integrity: Reduction of Risk Potential

A client has intra-arterial blood pressure monitoring after a myocardial infarction. The nurse notes the client's heart rate has increased from 88 to 110 beats/min, and the blood pressure dropped from 120/82 to 100/60 mm Hg. What action by the nurse is most appropriate? a. Allow the client to rest quietly. b. Assess the client for bleeding. c. Document the findings in the chart. d. Medicate the client for pain.

ANS: B A major complication related to intra-arterial blood pressure monitoring is hemorrhage from the insertion site. Since these vital signs are out of the normal range, are a change, and are consistent with blood loss, the nurse should assess the client for any bleeding associated with the arterial line. The nurse should document the findings after a full assessment. The client may or may not need pain medication and rest; the nurse first needs to rule out any emergent bleeding. DIF: Applying/Application REF: 771 KEY: Coronary artery disease| intra-arterial blood pressure monitoring| equipment safety| vital signs MSC: Integrated Process: Nursing Process: Assessment NOT: Client Needs Category: Physiological Integrity: Reduction of Risk Potential

A client received tissue plasminogen activator (t-PA) after a myocardial infarction and now is on an intravenous infusion of heparin. The client's spouse asks why the client needs this medication. What response by the nurse is best? a. "The t-PA didn't dissolve the entire coronary clot." b. "The heparin keeps that artery from getting blocked again." c. "Heparin keeps the blood as thin as possible for a longer time." d. "The heparin prevents a stroke from occurring as the t-PA wears off."

ANS: B After the original intracoronary clot has dissolved, large amounts of thrombin are released into the bloodstream, increasing the chance of the vessel reoccluding. The other statements are not accurate. Heparin is not a "blood thinner," although laypeople may refer to it as such. DIF: Understanding/Comprehension REF: 768 KEY: Coronary artery disease| thrombolytic agents| patient education MSC: Integrated Process: Teaching/Learning NOT: Client Needs Category: Physiological Integrity: Pharmacological and Parenteral Therapies

A client presents to the emergency department with a severely lacerated artery. What is the priority action for the nurse? a. Administer oxygen via non-rebreather mask. b. Ensure the client has a patent airway. c. Prepare to assist with suturing the artery. d. Start two large-bore IVs with normal saline.

ANS: B Airway always takes priority, followed by breathing and circulation. The nurse ensures the client has a patent airway prior to providing any other care measures. DIF: Applying/Application REF: 736 KEY: Critical rescue| primary survey| trauma MSC: Integrated Process: Nursing Process: Implementation NOT: Client Needs Category: Safe and Effective Care Environment: Management of Care

A client has hypertension and high risk factors for cardiovascular disease. The client is overwhelmed with the recommended lifestyle changes. What action by the nurse is best? a. Assess the client's support system. b. Assist in finding one change the client can control. c. Determine what stressors the client faces in daily life. d. Inquire about delegating some of the client's obligations.

ANS: B All options are appropriate when assessing stress and responses to stress. However, this client feels overwhelmed by the suggested lifestyle changes. Instead of looking at all the needed changes, the nurse should assist the client in choosing one the client feels optimistic about controlling. Once the client has mastered that change, he or she can move forward with another change. Determining support systems, daily stressors, and delegation opportunities does not directly impact the client's feelings of control. DIF: Applying/Application REF: 712 KEY: Hypertension| patient education| coping| psychosocial response MSC: Integrated Process: Nursing Process: Assessment NOT: Client Needs Category: Psychosocial Integrity

A nurse wants to provide community service that helps meet the goals of Healthy People 2020 (HP2020) related to cardiovascular disease and stroke. What activity would best meet this goal? a. Teach high school students heart-healthy living. b. Participate in blood pressure screenings at the mall. c. Provide pamphlets on heart disease at the grocery store. d. Set up an "Ask the nurse" booth at the pet store.

ANS: B An important goal of HP2020 is to increase the proportion of adults who have had their blood pressure measured within the preceding 2 years and can state whether their blood pressure was normal or high. Participating in blood pressure screening in a public spot will best help meet that goal. The other options are all appropriate but do not specifically help meet a goal. DIF: Applying/Application REF: 711 KEY: Hypertension| primary prevention MSC: Integrated Process: Nursing Process: Implementation NOT: Client Needs Category: Health Promotion and Maintenance

A nurse is caring for four clients. Which one should the nurse see first? a. Client who needs a beta blocker, and has a blood pressure of 92/58 mm Hg b. Client who had a first dose of captopril (Capoten) and needs to use the bathroom c. Hypertensive client with a blood pressure of 188/92 mm Hg d. Client who needs pain medication prior to a dressing change of a surgical wound

ANS: B Angiotensin-converting enzyme inhibitors such as captopril can cause hypotension, especially after the first dose. The nurse should see this client first to prevent falling if the client decides to get up without assistance. The two blood pressure readings are abnormal but not critical. The nurse should check on the client with higher blood pressure next to assess for problems related to the reading. The nurse can administer the beta blocker as standards state to hold it if the systolic blood pressure is below 90 mm Hg. The client who needs pain medication prior to the dressing change is not a priority over client safety and assisting the other client to the bathroom. DIF: Analyzing/Analysis REF: 716 KEY: Hypertension| angiotensin-converting enzyme (ACE) inhibitors| antihypertensive medications| patient safety MSC: Integrated Process: Nursing Process: Analysis NOT: Client Needs Category: Safe and Effective Care Environment: Management of Care

A client had a percutaneous transluminal coronary angioplasty for peripheral arterial disease. What assessment finding by the nurse indicates a priority outcome for this client has been met? a. Pain rated as 2/10 after medication b. Distal pulse on affected extremity 2+/4+ c. Remains on bedrest as directed d. Verbalizes understanding of procedure

ANS: B Assessing circulation distal to the puncture site is a critical nursing action. A pulse of 2+/4+ indicates good perfusion. Pain control, remaining on bedrest as directed after the procedure, and understanding are all important, but do not take priority over perfusion. DIF: Evaluating/Synthesis REF: 722 KEY: Peripheral vascular disease| perfusion| nursing assessment MSC: Integrated Process: Nursing Process: Evaluation NOT: Client Needs Category: Physiological Integrity: Reduction of Risk Potential

A nurse is assessing clients on a medical-surgical unit. Which client should the nurse identify as being at greatest risk for atrial fibrillation? a. A 45-year-old who takes an aspirin daily b. A 50-year-old who is post coronary artery bypass graft surgery c. A 78-year-old who had a carotid endarterectomy d. An 80-year-old with chronic obstructive pulmonary disease

ANS: B Atrial fibrillation occurs commonly in clients with cardiac disease and is a common occurrence after coronary artery bypass graft surgery. The other conditions do not place these clients at higher risk for atrial fibrillation. DIF: Applying/Application REF: 666 KEY: Health screening| cardiac electrical conduction MSC: Integrated Process: Nursing Process: Assessment NOT: Client Needs Category: Safe and Effective Care Environment: Management of Care

A nurse evaluates prescriptions for a client with chronic atrial fibrillation. Which medication should the nurse expect to find on this client's medication administration record to prevent a common complication of this condition? a. Sotalol (Betapace) b. Warfarin (Coumadin) c. Atropine (Sal-Tropine) d. Lidocaine (Xylocaine)

ANS: B Atrial fibrillation puts clients at risk for developing emboli. Clients at risk for emboli are treated with anticoagulants, such as heparin, enoxaparin, or warfarin. Sotalol, atropine, and lidocaine are not appropriate for this complication. DIF: Applying/Application REF: 667 KEY: Cardiac electrical conduction| medication MSC: Integrated Process: Nursing Process: Analysis NOT: Client Needs Category: Physiological Integrity: Pharmacological and Parenteral Therapies

A nurse cares for a client who has a heart rate averaging 56 beats/min with no adverse symptoms. Which activity modification should the nurse suggest to avoid further slowing of the heart rate? a. "Make certain that your bath water is warm." b. "Avoid straining while having a bowel movement." c. "Limit your intake of caffeinated drinks to one a day." d. "Avoid strenuous exercise such as running."

ANS: B Bearing down strenuously during a bowel movement is one type of Valsalva maneuver, which stimulates the vagus nerve and results in slowing of the heart rate. Such a response is not desirable in a person who has bradycardia. The other instructions are not appropriate for this condition. DIF: Applying/Application REF: 663 KEY: Functional ability MSC: Integrated Process: Nursing Process: Implementation NOT: Client Needs Category: Physiological Integrity: Basic Care and Comfort

A nurse teaches a client with diabetes mellitus and a body mass index of 42 who is at high risk for coronary artery disease. Which statement related to nutrition should the nurse include in this client's teaching? a. "The best way to lose weight is a high-protein, low-carbohydrate diet." b. "You should balance weight loss with consuming necessary nutrients." c. "A nutritionist will provide you with information about your new diet." d. "If you exercise more frequently, you won't need to change your diet."

ANS: B Clients at risk for cardiovascular diseases should follow the American Heart Association guidelines to combat obesity and improve cardiac health. The nurse should encourage the client to eat vegetables, fruits, unrefined whole-grain products, and fat-free dairy products while losing weight. High-protein food items are often high in fat and calories. Although the nutritionist can assist with client education, the nurse should include nutrition education and assist the client to make healthy decisions. Exercising and eating nutrient-rich foods are both important components in reducing cardiovascular risk. DIF: Applying/Application REF: 633 KEY: Nutrition| patient education MSC: Integrated Process: Teaching/Learning NOT: Client Needs Category: Physiological Integrity: Basic Care and Comfort

A client is receiving an infusion of alteplase (Activase) for an intra-arterial clot. The client begins to mumble and is disoriented. What action by the nurse takes priority? a. Assess the client's neurologic status. b. Notify the Rapid Response Team. c. Prepare to administer vitamin K. d. Turn down the infusion rate.

ANS: B Clients on fibrinolytic therapy are at high risk of bleeding. The sudden onset of neurologic signs may indicate the client is having a hemorrhagic stroke. The nurse does need to complete a thorough neurological examination, but should first call the Rapid Response Team based on the client's manifestations. The nurse notifies the Rapid Response Team first. Vitamin K is not the antidote for this drug. Turning down the infusion rate will not be helpful if the client is still receiving any of the drug. DIF: Applying/Application REF: 726 KEY: Critical rescue| Rapid Response Team| medical emergencies| fibrinolytic agents MSC: Integrated Process: Nursing Process: Implementation NOT: Client Needs Category: Safe and Effective Care Environment: Management of Care

A nurse assesses a client with atrial fibrillation. Which manifestation should alert the nurse to the possibility of a serious complication from this condition? a. Sinus tachycardia b. Speech alterations c. Fatigue d. Dyspnea with activity

ANS: B Clients with atrial fibrillation are at risk for embolic stroke. Evidence of embolic events includes changes in mentation, speech, sensory function, and motor function. Clients with atrial fibrillation often have a rapid ventricular response as a result. Fatigue is a nonspecific complaint. Clients with atrial fibrillation often have dyspnea as a result of the decreased cardiac output caused by the rhythm disturbance. DIF: Applying/Application REF: 666 KEY: Cardiac electrical conduction| vascular perfusion MSC: Integrated Process: Nursing Process: Assessment NOT: Client Needs Category: Physiological Integrity: Reduction of Risk Potential

A home health care nurse is visiting an older client who lives alone after being discharged from the hospital after a coronary artery bypass graft. What finding in the home most causes the nurse to consider additional referrals? a. Dirty carpets in need of vacuuming b. Expired food in the refrigerator c. Old medications in the kitchen d. Several cats present in the home

ANS: B Expired food in the refrigerator demonstrates a safety concern for the client and a possible lack of money to buy food. The nurse can consider a referral to Meals on Wheels or another home-based food program. Dirty carpets may indicate the client has no household help and is waiting for clearance to vacuum. Old medications can be managed by the home health care nurse and the client working collaboratively. Having pets is not a cause for concern. DIF: Applying/Application REF: 781 KEY: Home safety| referrals| coronary artery bypass graft MSC: Integrated Process: Communication and Documentation NOT: Client Needs Category: Safe and Effective Care Environment: Safety and Infection Control

A nurse assists with the cardioversion of a client experiencing acute atrial fibrillation. Which action should the nurse take prior to the initiation of cardioversion? a. Administer intravenous adenosine. b. Turn off oxygen therapy. c. Ensure a tongue blade is available. d. Position the client on the left side

ANS: B For safety during cardioversion, the nurse should turn off any oxygen therapy to prevent fire. The other interventions are not appropriate for a cardioversion. The client should be placed in a supine position. DIF: Remembering/Knowledge REF: 668 KEY: Assessment/diagnostic examination| safety MSC: Integrated Process: Nursing Process: Implementation NOT: Client Needs Category: Safe and Effective Care Environment: Safety and Infection Control

A nurse is in charge of the coronary intensive care unit. Which client should the nurse see first? a. Client on a nitroglycerin infusion at 5 mcg/min, not titrated in the last 4 hours b. Client who is 1 day post coronary artery bypass graft, blood pressure 180/100 mm Hg c. Client who is 1 day post percutaneous coronary intervention, going home this morning d. Client who is 2 days post coronary artery bypass graft, became dizzy this a.m. while walking

ANS: B Hypertension after coronary artery bypass graft surgery can be dangerous because it puts too much pressure on the suture lines and can cause bleeding. The charge nurse should see this client first. The client who became dizzy earlier should be seen next. The client on the nitroglycerin drip is stable. The client going home can wait until the other clients are cared for. DIF: Analyzing/Analysis REF: 777 KEY: Coronary artery disease| coronary artery bypass graft| collaboration MSC: Integrated Process: Nursing Process: Assessment NOT: Client Needs Category: Safe and Effective Care Environment: Management of Care

After assessing a client who is receiving an amiodarone intravenous infusion for unstable ventricular tachycardia, the nurse documents the findings and compares these with the previous assessment findings: Vital Signs Nursing Assessment Time: 0800 Temperature: 98° F Heart rate: 68 beats/min Blood pressure: 135/60 mm Hg Respiratory rate: 14 breaths/min Oxygen saturation: 96% Oxygen therapy: 2 L nasal cannula Time: 1000 Temperature: 98.2° F Heart rate: 50 beats/min Blood pressure: 132/57 mm Hg Respiratory rate: 16 breaths/min Oxygen saturation: 95% Oxygen therapy: 2 L nasal cannula Time: 0800 Client alert and oriented. Cardiac rhythm: normal sinus rhythm. Skin: warm, dry, and appropriate for race. Respirations equal and unlabored. Client denies shortness of breath and chest pain. Time: 1000 Client alert and oriented. Cardiac rhythm: sinus bradycardia. Skin: warm, dry, and appropriate for race. Respirations equal and unlabored. Client denies shortness of breath and chest pain. Client voids 420 mL of clear yellow urine. Based on the assessments, which action should the nurse take? a. Stop the infusion and flush the IV. b. Slow the amiodarone infusion rate. c. Administer IV normal saline. d. Ask the client to cough and deep breathe.

ANS: B IV administration of amiodarone may cause bradycardia and atrioventricular (AV) block. The correct action for the nurse to take at this time is to slow the infusion, because the client is asymptomatic and no evidence reveals AV block that might require pacing. Abruptly ceasing the medication could allow fatal dysrhythmias to occur. The administration of IV fluids and encouragement of coughing and deep breathing exercises are not indicated, and will not increase the client's heart rate. DIF: Applying/Application REF: 660 KEY: Cardiac electrical conduction| medication MSC: Integrated Process: Nursing Process: Implementation NOT: Client Needs Category: Physiological Integrity: Pharmacological and Parenteral Therapies

A client has hemodynamic monitoring after a myocardial infarction. What safety precaution does the nurse implement for this client? a. Document pulmonary artery wedge pressure (PAWP) readings and assess their trends. b. Ensure the balloon does not remain wedged. c. Keep the client on strict NPO status. d. Maintain the client in a semi-Fowler's position.

ANS: B If the balloon remains inflated, it can cause pulmonary infarction or rupture. The nurse should ensure the balloon remains deflated between PAWP readings. Documenting PAWP readings and assessing trends is an important nursing action related to hemodynamic monitoring, but is not specifically related to safety. The client does not have to be NPO while undergoing hemodynamic monitoring. Positioning may or may not affect readings. DIF: Applying/Application REF: 770 KEY: Coronary artery disease| hemodynamic monitoring| equipment safety MSC: Integrated Process: Nursing Process: Implementation NOT: Client Needs Category: Safe and Effective Care Environment: Safety and Infection Control

The nurse is caring for a client with a chest tube after a coronary artery bypass graft. The drainage slows significantly. What action by the nurse is most important? a. Increase the setting on the suction. b. Notify the provider immediately. c. Re-position the chest tube. d. Take the tubing apart to assess for clots.

ANS: B If the drainage in the chest tube decreases significantly and dramatically, the tube may be blocked by a clot. This could lead to cardiac tamponade. The nurse should notify the provider immediately. The nurse should not independently increase the suction, re-position the chest tube, or take the tubing apart. DIF: Applying/Application REF: 778 KEY: Coronary artery bypass graft| critical rescue| chest tubes| cardiovascular system MSC: Integrated Process: Communication and Documentation NOT: Client Needs Category: Safe and Effective Care Environment: Management of Care

A nurse cares for a client with an intravenous temporary pacemaker for bradycardia. The nurse observes the presence of a pacing spike but no QRS complex on the client's electrocardiogram. Which action should the nurse take next? a. Administer intravenous diltiazem (Cardizem). b. Assess vital signs and level of consciousness. c. Administer sublingual nitroglycerin. d. Assess capillary refill and temperature.

ANS: B In temporary pacing, the wires are threaded onto the epicardial surface of the heart and exit through the chest wall. The pacemaker spike should be followed immediately by a QRS complex. Pacing spikes seen without subsequent QRS complexes imply loss of capture. If there is no capture, then there is no ventricular depolarization and contraction. The nurse should assess for cardiac output via vital signs and level of consciousness. The other interventions would not determine if the client is tolerating the loss of capture. DIF: Applying/Application REF: 664 KEY: Cardiac electrical conduction MSC: Integrated Process: Nursing Process: Assessment NOT: Client Needs Category: Physiological Integrity: Reduction of Risk Potential

A client has been diagnosed with hypertension but does not take the antihypertensive medications because of a lack of symptoms. What response by the nurse is best? a. "Do you have trouble affording your medications?" b. "Most people with hypertension do not have symptoms." c. "You are lucky; most people get severe morning headaches." d. "You need to take your medicine or you will get kidney failure."

ANS: B Most people with hypertension are asymptomatic, although a small percentage do have symptoms such as headache. The nurse should explain this to the client. Asking about paying for medications is not related because the client has already admitted nonadherence. Threatening the client with possible complications will not increase compliance. DIF: Understanding/Comprehension REF: 711 KEY: Hypertension| antihypertensive medications| medication adherence MSC: Integrated Process: Communication and Documentation NOT: Client Needs Category: Physiological Integrity: Physiological Adaptation

A client with coronary artery disease (CAD) asks the nurse about taking fish oil supplements. What response by the nurse is best? a. "Fish oil is contraindicated with most drugs for CAD." b. "The best source is fish, but pills have benefits too." c. "There is no evidence to support fish oil use with CAD." d. "You can reverse CAD totally with diet and supplements."

ANS: B Omega-3 fatty acids have shown benefit in reducing lipid levels, in reducing the incidence of sudden cardiac death, and for stabilizing atherosclerotic plaque. The best source is fish three times a week or some fish oil supplements. The other options are not accurate. DIF: Understanding/Comprehension REF: 761 KEY: Coronary artery disease| lipid-reducing agents| supplements| patient education MSC: Integrated Process: Teaching/Learning NOT: Client Needs Category: Health Promotion and Maintenance

A client is 4 hours postoperative after a femoropopliteal bypass. The client reports throbbing leg pain on the affected side, rated as 7/10. What action by the nurse takes priority? a. Administer pain medication as ordered. b. Assess distal pulses and skin color. c. Document the findings in the client's chart. d. Notify the surgeon immediately.

ANS: B Once perfusion has been restored or improved to an extremity, clients can often feel a throbbing pain due to the increased blood flow. However, it is important to differentiate this pain from ischemia. The nurse should assess for other signs of perfusion, such as distal pulses and skin color/temperature. Administering pain medication is done once the nurse determines the client's perfusion status is normal. Documentation needs to be thorough. Notifying the surgeon is not necessary. DIF: Applying/Application REF: 724 KEY: Peripheral vascular disease| pain assessment| nursing process assessment| postoperative nursing MSC: Integrated Process: Nursing Process: Assessment NOT: Client Needs Category: Safe and Effective Care Environment: Management of Care

A client has presented to the emergency department with an acute myocardial infarction (MI). What action by the nurse is best to meet The Joint Commission's Core Measures outcomes? a. Obtain an electrocardiogram (ECG) now and in the morning. b. Give the client an aspirin. c. Notify the Rapid Response Team. d. Prepare to administer thrombolytics.

ANS: B The Joint Commission's Core Measures set for acute MI require that aspirin is administered when a client with MI presents to the emergency department or when an MI occurs in the hospital. A rapid ECG is vital, but getting another one in the morning is not part of the Core Measures set. The Rapid Response Team is not needed if an emergency department provider is available. Thrombolytics may or may not be needed. DIF: Remembering/Knowledge REF: 766 KEY: Coronary artery disease| Core Measures| The Joint Commission MSC: Integrated Process: Nursing Process: Implementation NOT: Client Needs Category: Safe and Effective Care Environment: Management of Care

. A nurse cares for a client who has an 80% blockage of the right coronary artery (RCA) and is scheduled for bypass surgery. Which intervention should the nurse be prepared to implement while this client waits for surgery? a. Administration of IV furosemide (Lasix) b. Initiation of an external pacemaker c. Assistance with endotracheal intubation d. Placement of central venous access

ANS: B The RCA supplies the right atrium, the right ventricle, the inferior portion of the left ventricle, and the atrioventricular (AV) node. It also supplies the sinoatrial node in 50% of people. If the client totally occludes the RCA, the AV node would not function and the client would go into heart block, so emergency pacing should be available for the client. Furosemide, intubation, and central venous access will not address the primary complication of RCA occlusion, which is AV node malfunction. DIF: Applying/Application REF: 628 KEY: Coronary perfusion| assessment/diagnostic examination| cardiac electrical conduction MSC: Integrated Process: Nursing Process: Planning NOT: Client Needs Category: Physiological Integrity: Reduction of Risk Potential

After teaching a client who has an implantable cardioverter-defibrillator (ICD), a nurse assesses the client's understanding. Which statement by the client indicates a correct understanding of the teaching? a. "I should wear a snug-fitting shirt over the ICD." b. "I will avoid sources of strong electromagnetic fields." c. "I should participate in a strenuous exercise program." d. "Now I can discontinue my antidysrhythmic medication."

ANS: B The client being discharged with an ICD is instructed to avoid strong sources of electromagnetic fields. Clients should avoid tight clothing, which could cause irritation over the ICD generator. The client should be encouraged to exercise but should not engage in strenuous activities that cause the heart rate to meet or exceed the ICD cutoff point because the ICD can discharge inappropriately. The client should continue all prescribed medications. DIF: Applying/Application REF: 674 KEY: Cardiac electrical conduction MSC: Integrated Process: Teaching/Learning NOT: Client Needs Category: Health Promotion and Maintenance

The nurse is caring for a client on the medical-surgical unit who suddenly becomes unresponsive and has no pulse. The cardiac monitor shows the rhythm below: After calling for assistance and a defibrillator, which action should the nurse take next? a. Perform a pericardial thump. b. Initiate cardiopulmonary resuscitation (CPR). c. Start an 18-gauge intravenous line. d. Ask the client's family about code status.

ANS: B The client's rhythm is ventricular fibrillation. This is a lethal rhythm that is best treated with immediate defibrillation. While the nurse is waiting for the defibrillator to arrive, the nurse should start CPR. A pericardial thump is not a treatment for ventricular fibrillation. If the client does not already have an IV, other members of the team can insert one after defibrillation. The client's code status should already be known by the nurse prior to this event. DIF: Applying/Application REF: 671 KEY: Cardiac electrical conduction| medical emergency MSC: Integrated Process: Nursing Process: Assessment NOT: Client Needs Category: Physiological Integrity: Physiological Adaptation

The nurse is evaluating a 3-day diet history with a client who has an elevated lipid panel. What meal selection indicates the client is managing this condition well with diet? a. A 4-ounce steak, French fries, iceberg lettuce b. Baked chicken breast, broccoli, tomatoes c. Fried catfish, cornbread, peas d. Spaghetti with meat sauce, garlic bread

ANS: B The diet recommended for this client would be low in saturated fats and red meat, high in vegetables and whole grains (fiber), low in salt, and low in trans fat. The best choice is the chicken with broccoli and tomatoes. The French fries have too much fat and the iceberg lettuce has little fiber. The catfish is fried. The spaghetti dinner has too much red meat and no vegetables. DIF: Evaluating/Synthesis REF: 708 KEY: Nutrition| fiber| self-care MSC: Integrated Process: Nursing Process: Evaluation NOT: Client Needs Category: Health Promotion and Maintenance

A nurse cares for a client who is prescribed magnetic resonance imaging (MRI) of the heart. The client's health history includes a previous myocardial infarction and pacemaker implantation. Which action should the nurse take? a. Schedule an electrocardiogram just before the MRI. b. Notify the health care provider before scheduling the MRI. c. Call the physician and request a laboratory draw for cardiac enzymes. d. Instruct the client to increase fluid intake the day before the MRI.

ANS: B The magnetic fields of the MRI can deactivate the pacemaker. The nurse should call the health care provider and report that the client has a pacemaker so the provider can order other diagnostic tests. The client does not need an electrocardiogram, cardiac enzymes, or increased fluids. DIF: Applying/Application REF: 647 KEY: Assessment/diagnostic examination| patient safety MSC: Integrated Process: Nursing Process: Planning NOT: Client Needs Category: Safe and Effective Care Environment: Safety and Infection Control

A nurse is caring for four clients. Which client should the nurse assess first? a. Client with an acute myocardial infarction, pulse 102 beats/min b. Client who is 1 hour post angioplasty, has tongue swelling and anxiety c. Client who is post coronary artery bypass, chest tube drained 100 mL/hr d. Client who is post coronary artery bypass, potassium 4.2 mEq/L

ANS: B The post-angioplasty client with tongue swelling and anxiety is exhibiting manifestations of an allergic reaction that could progress to anaphylaxis. The nurse should assess this client first. The client with a heart rate of 102 beats/min may have increased oxygen demands but is just over the normal limit for heart rate. The two post coronary artery bypass clients are stable. DIF: Analyzing/Analysis REF: 774 KEY: Coronary artery disease| critical rescue| medical emergencies| hypersensitivities| allergic reaction MSC: Integrated Process: Nursing Process: Analysis NOT: Client Needs Category: Safe and Effective Care Environment: Management of Care

A nurse auscultated heart tones on an older adult client. Which action should the nurse take based on heart tones heard? (Click the media button to hear the audio clip.) a. Administer a diuretic. b. Document the finding. c. Decrease the IV flow rate. d. Evaluate the client's medications.

ANS: B The sound heard is an atrial gallop S4. An atrial gallop may be heard in older clients because of a stiffened ventricle. The nurse should document the finding, but no other intervention is needed at this time. DIF: Applying/Application REF: 639 KEY: Assessment/diagnostic examination MSC: Integrated Process: Nursing Process: Implementation NOT: Client Needs Category: Health Promotion and Maintenance

A client is in the hospital after suffering a myocardial infarction and has bathroom privileges. The nurse assists the client to the bathroom and notes the client's O2 saturation to be 95%, pulse 88 beats/min, and respiratory rate 16 breaths/min after returning to bed. What action by the nurse is best? a. Administer oxygen at 2 L/min. b. Allow continued bathroom privileges. c. Obtain a bedside commode. d. Suggest the client use a bedpan.

ANS: B This client's physiologic parameters did not exceed normal during and after activity, so it is safe for the client to continue using the bathroom. There is no indication that the client needs oxygen, a commode, or a bedpan. DIF: Applying/Application REF: 769 KEY: Coronary artery disease| activity intolerance| vital signs| nursing assessment MSC: Integrated Process: Nursing Process: Assessment NOT: Client Needs Category: Physiological Integrity: Reduction of Risk Potential

A client has a deep vein thrombosis (DVT). What comfort measure does the nurse delegate to the unlicensed assistive personnel (UAP)? a. Ambulate the client. b. Apply a warm moist pack. c. Massage the client's leg. d. Provide an ice pack.

ANS: B Warm moist packs will help with the pain of a DVT. Ambulation is not a comfort measure. Massaging the client's legs is contraindicated to prevent complications such as pulmonary embolism. Ice packs are not recommended for DVT. DIF: Understanding/Comprehension REF: 731 KEY: Thromboembolic event| deep vein thrombosis| comfort measures| nonpharmacologic comfort interventions| unlicensed assistive personnel (UAP) MSC: Integrated Process: Communication and Documentation NOT: Client Needs Category: Physiological Integrity: Basic Care and Comfort

A nurse obtains the health history of a client who is newly admitted to the medical unit. Which statement by the client should alert the nurse to the presence of edema? a. "I wake up to go to the bathroom at night." b. "My shoes fit tighter by the end of the day." c. "I seem to be feeling more anxious lately." d. "I drink at least eight glasses of water a day."

ANS: B Weight gain can result from fluid accumulation in the interstitial spaces. This is known as edema. The nurse should note whether the client feels that his or her shoes or rings are tight, and should observe, when present, an indentation around the leg where the socks end. The other answers do not describe edema. DIF: Applying/Application REF: 635 KEY: Heart failure| vascular perfusion MSC: Integrated Process: Nursing Process: Assessment NOT: Client Needs Category: Physiological Integrity: Physiological Adaptation

What nonpharmacologic comfort measures should the nurse include in the plan of care for a client with severe varicose veins? (Select all that apply.) a. Administering mild analgesics for pain b. Applying elastic compression stockings c. Elevating the legs when sitting or lying d. Reminding the client to do leg exercises e. Teaching the client about surgical options

ANS: B, C, D The three E's of care for varicose veins include elastic compression hose, exercise, and elevation. Mild analgesics are not a nonpharmacologic measure. Teaching about surgical options is not a comfort measure. DIF: Understanding/Comprehension REF: 736 KEY: Peripheral vascular disease| nonpharmacologic comfort measures MSC: Integrated Process: Nursing Process: Implementation NOT: Client Needs Category: Physiological Integrity: Basic Care and Comfort

A nursing student learns about modifiable risk factors for coronary artery disease. Which factors does this include? (Select all that apply.) a. Age b. Hypertension c. Obesity d. Smoking e. Stress

ANS: B, C, D, E Hypertension, obesity, smoking, and excessive stress are all modifiable risk factors for coronary artery disease. Age is a nonmodifiable risk factor. DIF: Remembering/Knowledge REF: 760 KEY: Coronary artery disease| lifestyle factors MSC: Integrated Process: Teaching/Learning NOT: Client Needs Category: Health Promotion and Maintenance

An emergency room nurse assesses a female client. Which assessment findings should alert the nurse to request a prescription for an electrocardiogram? (Select all that apply.) a. Hypertension b. Fatigue despite adequate rest c. Indigestion d. Abdominal pain e. Shortness of breath

ANS: B, C, E Women may not have chest pain with myocardial infarction, but may feel discomfort or indigestion. They often present with a triad of symptoms—indigestion or feeling of abdominal fullness, feeling of chronic fatigue despite adequate rest, and feeling unable to catch their breath. Frequently, women are not diagnosed and therefore are not treated adequately. Hypertension and abdominal pain are not associated with acute coronary syndrome. DIF: Applying/Application REF: 635 KEY: Cardiac electrical conduction MSC: Integrated Process: Nursing Process: Assessment NOT: Client Needs Category: Physiological Integrity: Physiological Adaptation

A nurse assesses a client who is recovering after a coronary catheterization. Which assessment findings in the first few hours after the procedure require immediate action by the nurse? (Select all that apply.) a. Blood pressure of 140/88 mm Hg b. Serum potassium of 2.9 mEq/L c. Warmth and redness at the site d. Expanding groin hematoma e. Rhythm changes on the cardiac monitor

ANS: B, D, E In the first few hours postprocedure, the nurse monitors for complications such as bleeding from the insertion site, hypotension, acute closure of the vessel, dye reaction, hypokalemia, and dysrhythmias. The client's blood pressure is slightly elevated but does not need immediate action. Warmth and redness at the site would indicate an infection, but this would not be present in the first few hours. DIF: Applying/Application REF: 643 KEY: Assessment/diagnostic examination MSC: Integrated Process: Nursing Process: Assessment NOT: Client Needs Category: Physiological Integrity: Reduction of Risk Potential

A client is 1 day postoperative after a coronary artery bypass graft. What nonpharmacologic comfort measures does the nurse include when caring for this client? (Select all that apply.) a. Administer pain medication before ambulating. b. Assist the client into a position of comfort in bed. c. Encourage high-protein diet selections. d. Provide complementary therapies such as music. e. Remind the client to splint the incision when coughing.

ANS: B, D, E Nonpharmacologic comfort measures can include positioning, complementary therapies, and splinting the chest incision. Medications are not nonpharmacologic. Food choices are not comfort measures. DIF: Applying/Application REF: 781 KEY: Coronary artery disease| nonpharmacologic comfort measures MSC: Integrated Process: Nursing Process: Implementation NOT: Client Needs Category: Physiological Integrity: Basic Care and Comfort

A client with a known abdominal aortic aneurysm reports dizziness and severe abdominal pain. The nurse assesses the client's blood pressure at 82/40 mm Hg. What actions by the nurse are most important? (Select all that apply.) a. Administer pain medication. b. Assess distal pulses every 10 minutes. c. Have the client sign a surgical consent. d. Notify the Rapid Response Team. e. Take vital signs every 10 minutes.

ANS: B, D, E This client may have a ruptured/rupturing aneurysm. The nurse should notify the Rapid Response team and perform frequent client assessments. Giving pain medication will lower the client's blood pressure even further. The nurse cannot have the client sign a consent until the physician has explained the procedure. DIF: Applying/Application REF: 727 KEY: Critical rescue| Rapid Response Team| medical emergencies MSC: Integrated Process: Nursing Process: Implementation NOT: Client Needs Category: Physiological Integrity: Physiological Adaptation

A client is receiving an infusion of tissue plasminogen activator (t-PA). The nurse assesses the client to be disoriented to person, place, and time. What action by the nurse is best? a. Assess the client's pupillary responses. b. Request a neurologic consultation. c. Stop the infusion and call the provider. d. Take and document a full set of vital signs.

ANS: C A change in neurologic status in a client receiving t-PA could indicate intracranial hemorrhage. The nurse should stop the infusion and notify the provider immediately. A full assessment, including pupillary responses and vital signs, occurs next. The nurse may or may not need to call a neurologist. DIF: Applying/Application REF: 768 KEY: Coronary artery disease| neurologic system| critical rescue| Rapid Response Team| thrombolytic agents MSC: Integrated Process: Nursing Process: Implementation NOT: Client Needs Category: Physiological Integrity: Pharmacological and Parenteral Therapies

A telemetry nurse assesses a client with third-degree heart block who has wide QRS complexes and a heart rate of 35 beats/min on the cardiac monitor. Which assessment should the nurse complete next? a. Pulmonary auscultation b. Pulse strength and amplitude c. Level of consciousness d. Mobility and gait stability

ANS: C A heart rate of 40 beats/min or less with widened QRS complexes could have hemodynamic consequences. The client is at risk for inadequate cerebral perfusion. The nurse should assess for level of consciousness, light-headedness, confusion, syncope, and seizure activity. Although the other assessments should be completed, the client's level of consciousness is the priority. DIF: Applying/Application REF: 670 KEY: Cardiac electrical conduction| vascular perfusion MSC: Integrated Process: Nursing Process: Assessment NOT: Client Needs Category: Physiological Integrity: Reduction of Risk Potential

A nurse assesses a client who is recovering after a left-sided cardiac catheterization. Which assessment finding requires immediate intervention? a. Urinary output less than intake b. Bruising at the insertion site c. Slurred speech and confusion d. Discomfort in the left leg

ANS: C A left-sided cardiac catheterization specifically increases the risk for a cerebral vascular accident. A change in neurologic status needs to be acted on immediately. Discomfort and bruising are expected at the site. If intake decreases, a client can become dehydrated because of dye excretion. The second intervention would be to increase the client's fluid status. Neurologic changes would take priority. DIF: Applying/Application REF: 644 KEY: Assessment/diagnostic examination| vascular perfusion MSC: Integrated Process: Nursing Process: Assessment NOT: Client Needs Category: Safe and Effective Care Environment: Management of Care

The provider requests the nurse start an infusion of an inotropic agent on a client. How does the nurse explain the action of these drugs to the client and spouse? a. "It constricts vessels, improving blood flow." b. "It dilates vessels, which lessens the work of the heart." c. "It increases the force of the heart's contractions." d. "It slows the heart rate down for better filling."

ANS: C A positive inotrope is a medication that increases the strength of the heart's contractions. The other options are not correct. DIF: Remembering/Knowledge REF: 772 KEY: Coronary artery disease| inotropic agents| patient education MSC: Integrated Process: Teaching/Learning NOT: Client Needs Category: Physiological Integrity: Pharmacological and Parenteral Therapies

A client in the cardiac stepdown unit reports severe, crushing chest pain accompanied by nausea and vomiting. What action by the nurse takes priority? a. Administer an aspirin. b. Call for an electrocardiogram (ECG). c. Maintain airway patency. d. Notify the provider.

ANS: C Airway always is the priority. The other actions are important in this situation as well, but the nurse should stay with the client and ensure the airway remains patent (especially if vomiting occurs) while another person calls the provider (or Rapid Response Team) and facilitates getting an ECG done. Aspirin will probably be administered, depending on the provider's prescription and the client's current medications. DIF: Applying/Application REF: 769 KEY: Coronary artery disease| critical rescue| medical emergencies MSC: Integrated Process: Nursing Process: Implementation NOT: Client Needs Category: Safe and Effective Care Environment: Management of Care

A nurse administers prescribed adenosine (Adenocard) to a client. Which response should the nurse assess for as the expected therapeutic response? a. Decreased intraocular pressure b. Increased heart rate c. Short period of asystole d. Hypertensive crisis

ANS: C Clients usually respond to adenosine with a short period of asystole, bradycardia, hypotension, dyspnea, and chest pain. Adenosine has no conclusive impact on intraocular pressure. DIF: Applying/Application REF: 662 KEY: Cardiac electrical conduction| medication MSC: Integrated Process: Nursing Process: Analysis NOT: Client Needs Category: Physiological Integrity: Pharmacological and Parenteral Therapies

A nurse cares for a client who is recovering from a myocardial infarction. The client states, "I will need to stop eating so much chili to keep that indigestion pain from returning." How should the nurse respond? a. "Chili is high in fat and calories; it would be a good idea to stop eating it." b. "The provider has prescribed an antacid for you to take every morning." c. "What do you understand about what happened to you?" d. "When did you start experiencing this indigestion?"

ANS: C Clients who experience myocardial infarction often respond with denial, which is a defense mechanism. The nurse should ask the client what he or she thinks happened, or what the illness means to him or her. The other responses do not address the client's misconception about recent pain and the cause of that pain. DIF: Applying/Application REF: 640 KEY: Coronary perfusion| coping MSC: Integrated Process: Teaching/Learning NOT: Client Needs Category: Psychosocial Integrity

A nurse cares for a client with atrial fibrillation who reports fatigue when completing activities of daily living. What interventions should the nurse implement to address this client's concerns? a. Administer oxygen therapy at 2 liters per nasal cannula. b. Provide the client with a sleeping pill to stimulate rest. c. Schedule periods of exercise and rest during the day. d. Ask unlicensed assistive personnel to help bathe the client

ANS: C Clients who have atrial fibrillation are at risk for decreased cardiac output and fatigue when completing activities of daily living. The nurse should schedule periods of exercise and rest during the day to decrease fatigue. The other interventions will not assist the client with self-care activities. DIF: Applying/Application REF: 658 KEY: Cardiac electrical conduction MSC: Integrated Process: Teaching/Learning NOT: Client Needs Category: Physiological Integrity: Basic Care and Comfort

The nurse asks a client who has experienced ventricular dysrhythmias about substance abuse. The client asks, "Why do you want to know if I use cocaine?" How should the nurse respond? a. "Substance abuse puts clients at risk for many health issues." b. "The hospital requires that I ask you about cocaine use." c. "Clients who use cocaine are at risk for fatal dysrhythmias." d. "We can provide services for cessation of substance abuse."

ANS: C Clients who use cocaine or illicit inhalants are particularly at risk for potentially fatal dysrhythmias. The other responses do not adequately address the client's question. DIF: Remembering/Knowledge REF: 657 KEY: Cardiac electrical conduction| substance abuse MSC: Integrated Process: Nursing Process: Assessment NOT: Client Needs Category: Psychosocial Integrity

A nurse is assessing an obese client in the clinic for follow-up after an episode of deep vein thrombosis. The client has lost 20 pounds since the last visit. What action by the nurse is best? a. Ask if the weight loss was intended. b. Encourage a high-protein, high-fiber diet. c. Measure for new compression stockings. d. Review a 3-day food recall diary.

ANS: C Compression stockings must fit correctly in order to work. After losing a significant amount of weight, the client should be re-measured and new stockings ordered if needed. The other options are appropriate, but not the most important. DIF: Applying/Application REF: 731 KEY: Thromboembolic event| deep vein thrombosis| nursing assessment MSC: Integrated Process: Nursing Process: Assessment NOT: Client Needs Category: Physiological Integrity: Physiological Adaptation

A student nurse asks what "essential hypertension" is. What response by the registered nurse is best? a. "It means it is caused by another disease." b. "It means it is 'essential' that it be treated." c. "It is hypertension with no specific cause." d. "It refers to severe and life-threatening hypertension."

ANS: C Essential hypertension is the most common type of hypertension and has no specific cause such as an underlying disease process. Hypertension that is due to another disease process is called secondary hypertension. A severe, life-threatening form of hypertension is malignant hypertension. DIF: Understanding/Comprehension REF: 710 KEY: Hypertension| pathophysiology| patient education MSC: Integrated Process: Teaching/Learning NOT: Client Needs Category: Physiological Integrity: Physiological Adaptation

A client has an intra-arterial blood pressure monitoring line. The nurse notes bright red blood on the client's sheets. What action should the nurse perform first? a. Assess the insertion site. b. Change the client's sheets. c. Put on a pair of gloves. d. Assess blood pressure.

ANS: C For the nurse's safety, he or she should put on a pair of gloves to prevent blood exposure. The other actions are appropriate as well, but first the nurse must don a pair of gloves. DIF: Applying/Application REF: 771 KEY: Standard Precautions| infection control| intra-arterial blood pressure monitoring| staff safety MSC: Integrated Process: Nursing Process: Implementation NOT: Client Needs Category: Safe and Effective Care Environment: Safety and Infection Control

A nurse assesses an older adult client who is experiencing a myocardial infarction. Which clinical manifestation should the nurse expect? a. Excruciating pain on inspiration b. Left lateral chest wall pain c. Disorientation and confusion d. Numbness and tingling of the arm

ANS: C In older adults, disorientation or confusion may be the major manifestation of myocardial infarction caused by poor cardiac output. Pain manifestations and numbness and tingling of the arm could also be related to the myocardial infarction. However, the nurse should be more concerned about the new onset of disorientation or confusion caused by decreased perfusion. DIF: Applying/Application REF: 637 KEY: Coronary perfusion| older adult MSC: Integrated Process: Nursing Process: Assessment NOT: Client Needs Category: Physiological Integrity: Physiological Adaptation

A nurse assesses a client 2 hours after a cardiac angiography via the left femoral artery. The nurse notes that the left pedal pulse is weak. Which action should the nurse take? a. Elevate the leg and apply a sandbag to the entrance site. b. Increase the flow rate of intravenous fluids. c. Assess the color and temperature of the left leg. d. Document the finding as "left pedal pulse of +1/4."

ANS: C Loss of a pulse distal to an angiography entry site is serious, indicating a possible arterial obstruction. The pulse may be faint because of edema. The left pulse should be compared with the right, and pulses should be compared with previous assessments, especially before the procedure. Assessing color (pale, cyanosis) and temperature (cool, cold) will identify a decrease in circulation. Once all peripheral and vascular assessment data are acquired, the primary health care provider should be notified. Simply documenting the findings is inappropriate. The leg should be positioned below the level of the heart or dangling to increase blood flow to the distal portion of the leg. Increasing intravenous fluids will not address the client's problem. DIF: Applying/Application REF: 644 KEY: Assessment/diagnostic examination| vascular perfusion MSC: Integrated Process: Nursing Process: Assessment NOT: Client Needs Category: Safe and Effective Care Environment: Management of Care

A nurse assesses an older adult client who has multiple chronic diseases. The client's heart rate is 48 beats/min. Which action should the nurse take first? a. Document the finding in the chart. b. Initiate external pacing. c. Assess the client's medications. d. Administer 1 mg of atropine.

ANS: C Pacemaker cells in the conduction system decrease in number as a person ages, resulting in bradycardia. The nurse should check the medication reconciliation for medications that might cause such a drop in heart rate, then should inform the health care provider. Documentation is important, but it is not the priority action. The heart rate is not low enough for atropine or an external pacemaker to be needed. DIF: Applying/Application REF: 633 KEY: Medication| health screening MSC: Integrated Process: Nursing Process: Assessment NOT: Client Needs Category: Safe and Effective Care Environment: Management of Care

A client had an acute myocardial infarction. What assessment finding indicates to the nurse that a significant complication has occurred? a. Blood pressure that is 20 mm Hg below baseline b. Oxygen saturation of 94% on room air c. Poor peripheral pulses and cool skin d. Urine output of 1.2 mL/kg/hr for 4 hours

ANS: C Poor peripheral pulses and cool skin may be signs of impending cardiogenic shock and should be reported immediately. A blood pressure drop of 20 mm Hg is not worrisome. An oxygen saturation of 94% is just slightly below normal. A urine output of 1.2 mL/kg/hr for 4 hours is normal. DIF: Remembering/Knowledge REF: 772 KEY: Coronary artery disease| critical rescue| nursing assessment MSC: Integrated Process: Nursing Process: Analysis NOT: Client Needs Category: Physiological Integrity: Reduction of Risk Potential

A client presents to the emergency department with an acute myocardial infarction (MI) at 1500 (3:00 PM). The facility has 24-hour catheterization laboratory abilities. To meet The Joint Commission's Core Measures set, by what time should the client have a percutaneous coronary intervention performed? a. 1530 (3:30 PM) b. 1600 (4:00 PM) c. 1630 (4:30 PM) d. 1700 (5:00 PM)

ANS: C The Joint Commission's Core Measures set for MI includes percutaneous coronary intervention within 90 minutes of diagnosis of myocardial infarction. Therefore, the client should have a percutaneous coronary intervention performed no later than 1630 (4:30 PM). DIF: Remembering/Knowledge REF: 774 KEY: Coronary artery disease| Core Measures| The Joint Commission MSC: Integrated Process: Communication and Documentation NOT: Client Needs Category: Safe and Effective Care Environment: Management of Care

A nurse assesses clients on a medical-surgical unit. Which client should the nurse identify as having the greatest risk for cardiovascular disease? a. An 86-year-old man with a history of asthma b. A 32-year-old Asian-American man with colorectal cancer c. A 45-year-old American Indian woman with diabetes mellitus d. A 53-year-old postmenopausal woman who is on hormone therapy

ANS: C The incidence of coronary artery disease and hypertension is higher in American Indians than in whites or Asian Americans. Diabetes mellitus increases the risk for hypertension and coronary artery disease in people of any race or ethnicity. Asthma, colorectal cancer, and hormone therapy do not increase risk for cardiovascular disease. DIF: Understanding/Comprehension REF: 632 KEY: Health screening MSC: Integrated Process: Nursing Process: Assessment NOT: Client Needs Category: Safe and Effective Care Environment: Management of Care

A nurse prepares a client for coronary artery bypass graft surgery. The client states, "I am afraid I might die." How should the nurse respond? a. "This is a routine surgery and the risk of death is very low." b. "Would you like to speak with a chaplain prior to surgery?" c. "Tell me more about your concerns about the surgery." d. "What support systems do you have to assist you?"

ANS: C The nurse should discuss the client's feelings and concerns related to the surgery. The nurse should not provide false hope or push the client's concerns off on the chaplain. The nurse should address support systems after addressing the client's current issue. DIF: Applying/Application REF: 647 KEY: Assessment/diagnostic examination| coping| anxiety MSC: Integrated Process: Teaching/Learning NOT: Client Needs Category: Psychosocial Integrity

. A nurse performs an admission assessment on a 75-year-old client with multiple chronic diseases. The client's blood pressure is 135/75 mm Hg and oxygen saturation is 94% on 2 liters per nasal cannula. The nurse assesses the client's rhythm on the cardiac monitor and observes the reading shown below: Which action should the nurse take first? a. Begin external temporary pacing. b. Assess peripheral pulse strength. c. Ask the client what medications he or she takes. d. Administer 1 mg of atropine.

ANS: C This client is stable and therefore does not require any intervention except to determine the cause of the bradycardia. Bradycardia is often caused by medications. Clients who have multiple chronic diseases are often on multiple medications that can interact with each other. The nurse should assess the client's current medications first. DIF: Applying/Application REF: 658 KEY: Cardiac electrical conduction| medications| adverse effects MSC: Integrated Process: Nursing Process: Assessment NOT: Client Needs Category: Physiological Integrity: Pharmacological and Parenteral Therapies

Which statements by the client indicate good understanding of foot care in peripheral vascular disease? (Select all that apply.) a. "A good abrasive pumice stone will keep my feet soft." b. "I'll always wear shoes if I can buy cheap flip-flops." c. "I will keep my feet dry, especially between the toes." d. "Lotion is important to keep my feet smooth and soft." e. "Washing my feet in room-temperature water is best."

ANS: C, D, E Good foot care includes appropriate hygiene and injury prevention. Keeping the feet dry; wearing good, comfortable shoes; using lotion; washing the feet in room-temperature water; and cutting the nails straight across are all important measures. Abrasive material such as pumice stones should not be used. Cheap flip-flops may not fit well and won't offer much protection against injury. DIF: Evaluating/Synthesis REF: 725 KEY: Peripheral vascular disease| injury prevention| patient education MSC: Integrated Process: Nursing Process: Evaluation NOT: Client Needs Category: Safe and Effective Care Environment: Safety and Infection Control

A nursing student is caring for a client with an abdominal aneurysm. What action by the student requires the registered nurse to intervene? a. Assesses the client for back pain b. Auscultates over abdominal bruit c. Measures the abdominal girth d. Palpates the abdomen in four quadrants

ANS: D Abdominal aneurysms should never be palpated as this increases the risk of rupture. The registered nurse should intervene when the student attempts to do this. The other actions are appropriate. DIF: Applying/Application REF: 726 KEY: Aneurysms| nursing process assessment| supervision| abdominal assessment MSC: Integrated Process: Communication and Documentation NOT: Client Needs Category: Safe and Effective Care Environment: Safety and Infection Control

An emergency department nurse triages clients who present with chest discomfort. Which client should the nurse plan to assess first? a. A 42-year-old female who describes her pain as a dull ache with numbness in her fingers b. A 49-year-old male who reports moderate pain that is worse on inspiration c. A 53-year-old female who reports substernal pain that radiates to her abdomen d. A 58-year-old male who describes his pain as intense stabbing that spreads across his chest

ANS: D All clients who have chest pain should be assessed more thoroughly. To determine which client should be seen first, the nurse must understand common differences in pain descriptions. Intense stabbing, vise-like substernal pain that spreads through the client's chest, arms, jaw, back, or neck is indicative of a myocardial infarction. The nurse should plan to see this client first to prevent cardiac cell death. A dull ache with numbness in the fingers is consistent with anxiety. Pain that gets worse with inspiration is usually related to a pleuropulmonary problem. Pain that spreads to the abdomen is often associated with an esophageal-gastric problem, especially when this pain is experienced by a male client. Female clients may experience abdominal discomfort with a myocardial event. Although clients with anxiety, pleuropulmonary, and esophageal-gastric problems should be seen, they are not a higher priority than myocardial infarction. DIF: Applying/Application REF: 644 KEY: Assessment/diagnostic examination MSC: Integrated Process: Nursing Process: Planning NOT: Client Needs Category: Safe and Effective Care Environment: Management of Care

A nurse assesses a client who is scheduled for a cardiac catheterization. Which assessment should the nurse complete prior to this procedure? a. Client's level of anxiety b. Ability to turn self in bed c. Cardiac rhythm and heart rate d. Allergies to iodine-based agents

ANS: D Before the procedure, the nurse should ascertain whether the client has an allergy to iodine-containing preparations, such as seafood or local anesthetics. The contrast medium used during the procedure is iodine based. This allergy can cause a life-threatening reaction, so it is a high priority. Second, it is important for the nurse to assess anxiety, mobility, and baseline cardiac status. DIF: Remembering/Knowledge REF: 643 KEY: Assessment/diagnostic examination| allergies| patient safety MSC: Integrated Process: Nursing Process: Assessment NOT: Client Needs Category: Safe and Effective Care Environment: Safety and Infection Control

A nurse assesses a client after administering a prescribed beta blocker. Which assessment should the nurse expect to find? a. Blood pressure increased from 98/42 mm Hg to 132/60 mm Hg b. Respiratory rate decreased from 25 breaths/min to 14 breaths/min c. Oxygen saturation increased from 88% to 96% d. Pulse decreased from 100 beats/min to 80 beats/min

ANS: D Beta blockers block the stimulation of beta1-adrenergic receptors. They block the sympathetic (fight-or-flight) response and decrease the heart rate (HR). The beta blocker will decrease HR and blood pressure, increasing ventricular filling time. It usually does not have effects on beta2-adrenergic receptor sites. Cardiac output will drop because of decreased HR. DIF: Applying/Application REF: 630 KEY: Beta blocker| medication MSC: Integrated Process: Nursing Process: Assessment NOT: Client Needs Category: Physiological Integrity: Pharmacological and Parenteral Therapies

An older client with peripheral vascular disease (PVD) is explaining the daily foot care regimen to the family practice clinic nurse. What statement by the client may indicate a barrier to proper foot care? a. "I nearly always wear comfy sweatpants and house shoes." b. "I'm glad I get energy assistance so my house isn't so cold." c. "My daughter makes sure I have plenty of lotion for my feet." d. "My hands shake when I try to do things requiring coordination."

ANS: D Clients with PVD need to pay special attention to their feet. Toenails need to be kept short and cut straight across. The client whose hands shake may cause injury when trimming toenails. The nurse should refer this client to a podiatrist. Comfy sweatpants and house shoes are generally loose and not restrictive, which is important for clients with PVD. Keeping the house at a comfortable temperature makes it less likely the client will use alternative heat sources, such as heating pads, to stay warm. The client should keep the feet moist and soft with lotion. DIF: Analyzing/Analysis REF: 725 KEY: Peripheral vascular disease| self-care| home safety MSC: Integrated Process: Nursing Process: Analysis NOT: Client Needs Category: Health Promotion and Maintenance

A nurse assesses a client's electrocardiograph tracing and observes that not all QRS complexes are preceded by a P wave. How should the nurse interpret this observation? a. The client has hyperkalemia causing irregular QRS complexes. b. Ventricular tachycardia is overriding the normal atrial rhythm. c. The client's chest leads are not making sufficient contact with the skin. d. Ventricular and atrial depolarizations are initiated from different sites.

ANS: D Normal rhythm shows one P wave preceding each QRS complex, indicating that all depolarization is initiated at the sinoatrial node. QRS complexes without a P wave indicate a different source of initiation of depolarization. This finding on an electrocardiograph tracing is not an indication of hyperkalemia, ventricular tachycardia, or disconnection of leads. DIF: Understanding/Comprehension REF: 649 KEY: Cardiac electrical conduction MSC: Integrated Process: Nursing Process: Analysis NOT: Client Needs Category: Physiological Integrity: Physiological Adaptation

A client undergoing hemodynamic monitoring after a myocardial infarction has a right atrial pressure of 0.5 mm Hg. What action by the nurse is most appropriate? a. Level the transducer at the phlebostatic axis. b. Lay the client in the supine position. c. Prepare to administer diuretics. d. Prepare to administer a fluid bolus.

ANS: D Normal right atrial pressures are from 1 to 8 mm Hg. Lower pressures usually indicate hypovolemia, so the nurse should prepare to administer a fluid bolus. The transducer should remain leveled at the phlebostatic axis. Positioning may or may not influence readings. Diuretics would be contraindicated. DIF: Applying/Application REF: 770 KEY: Coronary artery disease| hemodynamic monitoring| fluid and electrolyte imbalance MSC: Integrated Process: Nursing Process: Implementation NOT: Client Needs Category: Physiological Integrity: Physiological Adaptation

A nurse assesses a client's electrocardiogram (ECG) and observes the reading shown below: How should the nurse document this client's ECG strip? a. Ventricular tachycardia b. Ventricular fibrillation c. Sinus rhythm with premature atrial contractions (PACs) d. Sinus rhythm with premature ventricular contractions (PVCs)

ANS: D Sinus rhythm with PVCs has an underlying regular sinus rhythm with ventricular depolarization that sometimes precede atrial depolarization. Ventricular tachycardia and ventricular fibrillation rhythms would not have sinus beats present. Premature atrial contractions are atrial contractions initiated from another region of the atria before the sinus node initiates atrial depolarization. DIF: Applying/Application REF: 669 KEY: Cardiac electrical conduction| documentation MSC: Integrated Process: Communication and Documentation NOT: Client Needs Category: Physiological Integrity: Physiological Adaptation

A nurse cares for a client who has advanced cardiac disease and states, "I am having trouble sleeping at night." How should the nurse respond? a. "I will consult the provider to prescribe a sleep study to determine the problem." b. "You become hypoxic while sleeping; oxygen therapy via nasal cannula will help." c. "A continuous positive airway pressure, or CPAP, breathing mask will help you breathe at night." d. "Use pillows to elevate your head and chest while you are sleeping."

ANS: D The client is experiencing orthopnea (shortness of breath while lying flat). The nurse should teach the client to elevate the head and chest with pillows or sleep in a recliner. A sleep study is not necessary to diagnose this client. Oxygen and CPAP will not help a client with orthopnea. DIF: Understanding/Comprehension REF: 635 KEY: Heart failure| respiratory distress/failure| patient education MSC: Integrated Process: Teaching/Learning NOT: Client Needs Category: Physiological Integrity: Basic Care and Comfort

A nurse is assessing a client who had a myocardial infarction. Upon auscultating heart sounds, the nurse hears the following sound. What action by the nurse is most appropriate? (Click the media button to hear the audio clip.) a. Assess for further chest pain. b. Call the Rapid Response Team. c. Have the client sit upright. d. Listen to the client's lung sounds.

ANS: D The sound the nurse hears is an S3 heart sound, an abnormal sound that may indicate heart failure. The nurse should next assess the client's lung sounds. Assessing for chest pain is not directly related. There is no indication that the Rapid Response Team is needed. Having the client sit up will not change the heart sound. DIF: Applying/Application REF: 762 KEY: Coronary artery disease| respiratory assessment| respiratory system| nursing assessment MSC: Integrated Process: Nursing Process: Assessment NOT: Client Needs Category: Physiological Integrity: Reduction of Risk Potential

A student nurse is assessing the peripheral vascular system of an older adult. What action by the student would cause the faculty member to intervene? a. Assessing blood pressure in both upper extremities b. Auscultating the carotid arteries for any bruits c. Classifying capillary refill of 4 seconds as normal d. Palpating both carotid arteries at the same time

ANS: D The student should not compress both carotid arteries at the same time to avoid brain ischemia. Blood pressure should be taken and compared in both arms. Prolonged capillary refill is considered to be greater than 5 seconds in an older adult, so classifying refill of 4 seconds as normal would not require intervention. Bruits should be auscultated. DIF: Remembering/Knowledge REF: 707 KEY: Nursing assessment| neurologic system| neurologic assessment MSC: Integrated Process: Communication and Documentation NOT: Client Needs Category: Physiological Integrity: Reduction of Risk Potential

A nurse prepares to defibrillate a client who is in ventricular fibrillation. Which priority intervention should the nurse perform prior to defibrillating this client? a. Make sure the defibrillator is set to the synchronous mode. b. Administer 1 mg of intravenous epinephrine. c. Test the equipment by delivering a smaller shock at 100 joules. d. Ensure that everyone is clear of contact with the client and the bed

ANS: D To avoid injury, the rescuer commands that all personnel clear contact with the client or the bed and ensures their compliance before delivery of the shock. A precordial thump can be delivered when no defibrillator is available. Defibrillation is done in asynchronous mode. Equipment should not be tested before a client is defibrillated because this is an emergency procedure; equipment should be checked on a routine basis. Epinephrine should be administered after defibrillation. DIF: Applying/Application REF: 668 KEY: Cardiac electrical conduction| safety MSC: Integrated Process: Nursing Process: Implementation NOT: Client Needs Category: Safe and Effective Care Environment: Safety and Infection Control

The nurse is preparing to change a client's sternal dressing. What action by the nurse is most important? a. Assess vital signs. b. Don a mask and gown. c. Gather needed supplies. d. Perform hand hygiene.

ANS: D To prevent a sternal wound infection, the nurse washes hands or performs hand hygiene as a priority. Vital signs do not necessarily need to be assessed beforehand. A mask and gown are not needed. The nurse should gather needed supplies, but this is not the priority. DIF: Applying/Application REF: 776 KEY: Coronary artery disease| infection control| hand hygiene MSC: Integrated Process: Nursing Process: Implementation NOT: Client Needs Category: Safe and Effective Care Environment: Safety and Infection Control

The nurse is reviewing the lipid panel of a male client who has atherosclerosis. Which finding is most concerning? a. Cholesterol: 126 mg/dL b. High-density lipoprotein cholesterol (HDL-C): 48 mg/dL c. Low-density lipoprotein cholesterol (LDL-C): 122 mg/dL d. Triglycerides: 198 mg/dL

ANS: D Triglycerides in men should be below 160 mg/dL. The other values are appropriate for adult males. DIF: Remembering/Knowledge REF: 708 KEY: Laboratory values| lipid alterations MSC: Integrated Process: Nursing Process: Assessment NOT: Client Needs Category: Physiological Integrity: Reduction of Risk Potential

A nurse is preparing a client for a femoropopliteal bypass operation. What actions does the nurse delegate to the unlicensed assistive personnel (UAP)? (Select all that apply.) a. Administering preoperative medication b. Ensuring the consent is signed c. Marking pulses with a pen d. Raising the siderails on the bed e. Recording baseline vital signs

ANS: D, E The UAP can raise the siderails of the bed for client safety and take and record the vital signs. Administering medications, ensuring a consent is on the chart, and marking the pulses for later comparison should be done by the registered nurse. This is also often done by the postanesthesia care nurse and is part of the hand-off report. DIF: Applying/Application REF: 723 KEY: Delegation| preoperative nursing| patient safety| unlicensed assistive personnel (UAP) MSC: Integrated Process: Communication and Documentation NOT: Client Needs Category: Safe and Effective Care Environment: Management of Care

A client has a bone marrow biopsy performed. What is the priority postprocedure nursing action? a. Inspect the site for ecchymosis b. Apply pressure to the biopsy site c. Send the biopsy specimens to the laboratory d. Teach the client to avoid vigorous activity

B

The nurse is assessing an adult client's endurance in performing activities of daily living (ADLs). What question would the nurse ask the client? a. "Can you prepare your own meals every day?" b. "How is your energy level compared with last year?" c. "Has your weight changed by 5 pounds (2.3 kg) or more this year?" d. "What medications do you take daily, weekly, and monthly?"

B

The nurse is starting the shift by making rounds. Which client would the nurse assess first? a. A 52-year-old who just had a bone marrow aspiration and is requesting pain medication b. A 59-year-old who has a nosebleed and is receiving heparin to treat a pulmonary embolism c. A 47-year-old who had a Rumpel-Leede test and asks the nurse to "look at the bruises on my arm" d. A 42-year-old with a diagnosis of anemia who reports shortness of breath when ambulating down the hallway

B

The nurse is teaching a client who is preparing for discharge after a bone marrow aspiration. The nurse provides which discharge instructions to the client? a. "Inspect the site for bleeding every 4 to 6 hours." b. "Place an ice pack over the site to reduce the bruising." c. "Avoid contact sports or activity that may traumatize the site for 24 hours." d. "Take a mild analgesic, such as two aspirin, for pain or discomfort at the site."

B

A client is scheduled for a bone marrow aspiration. What is the priority nursing action before this procedure is performed? a. Hold the client's hand and ask about concerns. b. Review the client's platelet (thrombocyte) count. c. Verify that the client has given informed consent. d. Clean the biopsy site with an antiseptic or povidone-iodine (Betadine).

C

A client on anticoagulant therapy is being discharged. Which statement by the client indicates an understanding of the anticoagulants drug action? a. "It will thin my blood." b. "It is used to dissolve blood clots." c. "It should prevent my blood from clotting." d. "It might cause me to get injured more often."

C

A client with anemia asks the nurse, "Do most people have the same number of red blood cells?" Which is the nurse's best response to the client? a. "Yes, they do." b. "No, they don't." c. "The number varies with gender, age, and general health." d. "You have fewer red blood cells because you have anemia."

C

A newly admitted client has an elevated reticulocyte count. Which condition does the nurse suspect in this client? a. Leukemia b. Aplastic anemia c. Hemolytic anemia d. Infectious process

C

After reviewing the laboratory test results, the nurse calls the primary care provider about which client? a. A 52-year-old who had a hemorrhage with a reticulocyte count of 0.8% b. A 49-year-old with hemophilia and a platelet count of 150,000/mm3 (150 × 109/L) c. A 46-year-old with a fever and a white blood cell (WBC) count of 1500/mm3 (1.5 × 109/L) d. A 44-year-old prescribed warfarin (Coumadin) with an international normalized ratio (INR) of 3.0

C

The nurse is assessing a client for hematologic risks. Which health history question would the nurse ask to determine if the risk cannot be reduced or eliminated? a. "Where do you work?" b. "Tell me what you eat in a day." c. "Does anyone in your family bleed a lot?" d. "Do you seem to have excessive bleeding or bruising?"

C

The nurse is assessing the nutritional status of a client with anemia. How does the nurse obtain information about the client's diet? a. Uses a prepared list and finds out the client's food preferences b. Asks the client to rate his or her diet on a scale of 1 (poor) to 10 (excellent) c. Has the client write down everything he or she has eaten for the past week d. Determines who prepares the client's meals and plans an interview with him or her

C

The nurse is caring for a group of hospitalized clients. Which client is at highest risk for infection and sepsis? a. A client with hemolytic anemia b. A client with cirrhosis of the liver c. A client who had an emergency splenectomy d. A client with recently diagnosed sickle cell anemia

C

The nurse is reviewing complete blood count (CBC) data for a 76-year-old client. Which decreased laboratory value would be of greatest concern to the nurse because it is not age-related? a. Hemoglobin level b. Red blood cell (RBC) count c. Platelet (thrombocyte) count d. White blood cell (WBC) response

C

The nurse is teaching a client about what to expect during a bone marrow biopsy. Which statement by the nurse accurately describes the procedure? a. "The doctor will place a small needle in your back and will withdraw some fluid." b. "You will be sedated during the procedure, so you will not be aware of anything." c. "You may experience a crunching sound or a scraping sensation as the needle punctures your bone." d. "You will be alone because the procedure is sterile; we cannot allow additional people to contaminate the area."

C

Which task does the nurse delegate to unlicensed assistive personnel (UAP)? a. Refer a client with a daily alcohol consumption of 12 beers for counseling b. Obtain a partial thromboplastin time from a saline lock on a client with a pulmonary embolism c. Report any bleeding noted when catheter care is given to a client with a history of hemophilia d. Perform a capillary fragility test to check vascular hemostatic function on a client with liver failure

C

A client with a low platelet count asks the nurse, "Why are platelets important?" Which statement is the nurse's best response? a. "Platelets will make your blood clot." b. "Your platelets finish the clotting process." c. "Blood clotting is prevented by your platelets." d. "The clotting process begins with your platelets."

D

A client with anemia asks the nurse, "Why am I feeling tired all the time?" What is the nurse's best response? a. "You are not getting enough iron." b. "When you are sick you need to rest more." c. "How many hours are you sleeping at night?" d. "Your cells are delivering less oxygen than you need."

D

Which task does the nurse delegate to unlicensed assistive personnel (UAP) who is assisting with the care of a female client with anemia? a. Monitor the oral mucosa for pallor, bleeding, or ulceration b. Ask about the amount of blood loss with each menstrual period c. Check for sternal tenderness while applying fingertip pressure d. Count the respiratory rate before and after ambulating 20 feet (6 m)

D

The nurse is performing an assessment on a client with anemia. What are the typical clinical manifestations of anemia? (Select all that apply.) Select all that apply. a. Fatigue b. Decreased breath sounds c. Pallor d. Tachycardia e. Dyspnea on exertion f. Elevated temperature

a, c, d, e, The typical clinical manifestations of anemia are: pallor, fatigue, tachycardia, and dyspnea on exertion. Lowered O2 levels deliver less oxygen to all cells, making clients with anemia pale—especially their ears, nail beds, palms, and conjunctivae and around the mouth. Fatigue is a classic symptom of anemia because lowered O2 levels contribute to a faster pulse (i.e., cardiac rate) and tend to "wear out" a client's energy. Difficulty breathing—especially with activity—is common with anemia. Lower levels of hemoglobin carry less O2 to the cells of the body. Respiratory problems with anemia do not include changes in breath sounds. Skin is cool to the touch, and an intolerance to cold is noted. Elevated temperature would signify something additional, such as infection.

A client in sickle cell crisis is dehydrated and in the emergency department. The nurse plans to start an IV. Which fluid choice is best? a. 0.45% normal saline b. 0.9% normal saline c. Dextrose 50% (D50) d. Lactated Ringer's solution

a. 0.45% normal saline

A client presents to the emergency department in sickle cell crisis. What intervention by the nurse takes priority? a. Administer oxygen. b. Apply an oximetry probe. c. Give pain medication. d. Start an IV line.

a. Administer oxygen.

A client has heparin-induced thrombocytopenia (HIT). The student nurse asks how this is treated. About what drugs does the nurse instructor teach? (Select all that apply.) a. Argatroban (Argatroban) b. Bivalirudin (Angiomax) c. Clopidogrel (Plavix) d. Lepirudin (Refludan) e. Methylprednisolone (Solu-Medrol)

a. Argatroban (Argatroban) b. Bivalirudin (Angiomax) d. Lepirudin (Refludan)

A nurse is preparing to administer a blood transfusion to an older adult. Understanding age-related changes, what alterations in the usual protocol are necessary for the nurse to implement? (Select all that apply.) a. Assess vital signs more often. b. Hold other IV fluids running. c. Premedicate to prevent reactions. d. Transfuse smaller bags of blood. e. Transfuse each unit over 8 hours.

a. Assess vital signs more often. b. Hold other IV fluids running.

A client has frequent hospitalizations for leukemia and is worried about functioning as a parent to four small children. What action by the nurse would be most helpful? a. Assist the client to make "sick day" plans for household responsibilities. b. Determine if there are family members or friends who can help the client. c. Help the client inform friends and family that they will have to help out. d. Refer the client to a social worker in order to investigate respite child care.

a. Assist the client to make "sick day" plans for household responsibilities.

A client has been admitted after sustaining a humerus fracture that occurred when picking up the family cat. What test result would the nurse correlate to this condition? a. Bence-Jones protein in urine b. Epstein-Barr virus: positive c. Hemoglobin: 18 mg/dL d. Red blood cell count: 8.2/mm3

a. Bence-Jones protein in urine

A client has a platelet count of 9000/mm3. The nurse finds the client confused and mumbling. What action takes priority? a. Calling the Rapid Response Team b. Delegating taking a set of vital signs c. Instituting bleeding precautions d. Placing the client on bedrest

a. Calling the Rapid Response Team

A student studying leukemias learns the risk factors for developing this disorder. Which risk factors does this include? (Select all that apply.) a. Chemical exposure b. Genetically modified foods c. Ionizing radiation exposure d. Vaccinations e. Viral infections

a. Chemical exposure c. Ionizing radiation exposure e. Viral infections

A nurse is caring for four clients with leukemia. After hand-off report, which client should the nurse see first? a. Client who had two bloody diarrhea stools this morning b. Client who has been premedicated for nausea prior to chemotherapy c. Client with a respiratory rate change from 18 to 22 breaths/min d. Client with an unchanged lesion to the lower right lateral malleolus

a. Client who had two bloody diarrhea stools this morning

A nurse caring for a client with sickle cell disease (SCD) reviews the client's laboratory work. Which finding should the nurse report to the provider? a. Creatinine: 2.9 mg/dL b. Hematocrit: 30% c. Sodium: 147 mEq/L d. White blood cell count: 12,000/mm3

a. Creatinine: 2.9 mg/dL

A nurse working with clients with sickle cell disease (SCD) teaches about self-management to prevent exacerbations and sickle cell crises. What factors should clients be taught to avoid? (Select all that apply.) a. Dehydration b. Exercise c. Extreme stress d. High altitudes e. Pregnancy

a. Dehydration c. Extreme stress d. High altitudes e. Pregnancy

The nurse is caring for a client with leukemia who has the priority problem of fatigue. What action by the client best indicates that an important goal for this problem has been met? a. Doing activities of daily living (ADLs) using rest periods b. Helping plan a daily activity schedule c. Requesting a sleeping pill at night d. Telling visitors to leave when fatigued

a. Doing activities of daily living (ADLs) using rest periods

A student nurse is learning about blood transfusion compatibilities. What information does this include? (Select all that apply.) a. Donor blood type A can donate to recipient blood type AB. b. Donor blood type B can donate to recipient blood type O. c. Donor blood type AB can donate to anyone. d. Donor blood type O can donate to anyone. e. Donor blood type A can donate to recipient blood type B.

a. Donor blood type A can donate to recipient blood type AB. d. Donor blood type O can donate to anyone.

A client has Crohn's disease. What type of anemia is this client most at risk for developing? a. Folic acid deficiency b. Fanconi's anemia c. Hemolytic anemia d. Vitamin B12 anemia

a. Folic acid deficiency

A client hospitalized with sickle cell crisis frequently asks for opioid pain medications, often shortly after receiving a dose. The nurses on the unit believe the client is drug seeking. When the client requests pain medication, what action by the nurse is best? a. Give the client pain medication if it is time for another dose. b. Instruct the client not to request pain medication too early. c. Request the provider leave a prescription for a placebo. d. Tell the client it is too early to have more pain medication.

a. Give the client pain medication if it is time for another dose.

A student nurse is helping a registered nurse with a blood transfusion. Which actions by the student are most appropriate? (Select all that apply.) a. Hanging the blood product using normal saline and a filtered tubing set b. Taking a full set of vital signs prior to starting the blood transfusion c. Telling the client someone will remain at the bedside for the first 5 minutes d. Using gloves to start the client's IV if needed and to handle the blood product e. Verifying the client's identity, and checking blood compatibility and expiration time

a. Hanging the blood product using normal saline and a filtered tubing set b. Taking a full set of vital signs prior to starting the blood transfusion d. Using gloves to start the client's IV if needed and to handle the blood product

Which client statement indicates in-home stem cell transplantation is not a viable option? a. I was a nurse, so I can take care of myself b. I don't feel strong enough, but my wife said she would help c. We live 5 miles from the hospital d. I will have lots of medicine to take

a. I was a nurse so I can take care of myself The client statement that indicates that in-home stem cell transplantation is not a viable option is "I was a nurse, so I can take care of myself." Stem cell transplantation in the home setting requires support, assistance, and coordination from others. The client cannot manage this type of care on his own. It is acceptable for the client's spouse to support the client undergoing this procedure. It is not unexpected for the client to be taking several prescriptions. Five miles is an acceptable distance from the hospital, in case of emergency.

A client recovering from a sickle cell crisis is to be discharged. The nurse says to the client, "You and all clients with sickle cell disease are at risk for infection because of your decreased spleen function. For this reason, you will most likely be prescribed an antibiotic before discharge." Which drug does the nurse anticipate the primary health care provider (PHCP) will prescribe? a. Penicillin V (Pen-V K) b. Gentamicin (Garamycin) c. Cefaclor (Ceclor) d. Vancomycin (Vancocin)

a. Penicillin V ( Pen-V K) The nurse expects the PHCP to prescribe Penicillin V for a client recovering from sickle cell crisis who is at risk for infection. Prophylactic therapy with twice-daily oral penicillin reduces the incidence of pneumonia and other streptococcal infections and is the correct drug to use. It is a standard protocol for long-term prophylactic use in clients with sickle cell disease. Cefaclor (Ceclor) and vancomycin (Vancocin) are antibiotics more specific for short-term use and would be inappropriate for this client. Gentamicin (Garamycin) is a drug that can cause liver and kidney damage with long-term use.

The nurse is assessing several clients who are receiving transfusions of blood components. Which assessment finding requires immediate action by the nurse? a. Respiratory rate of 36 breaths/min in a client receiving red blood cells b. Temperature of 99.1°F (37.3°C) for a client with a platelet transfusion c. Sleepiness in a client who received diphenhydramine (Benadryl) as a premedication d. A partial thromboplastin time (PTT) that is 1.2 times normal in a client who received a transfusion of fresh-frozen plasma (FFP)

a. Respiratory rate of 36 breaths/min in a client receiving red blood cells The assessment finding that requires immediate action by the nurse is a respiratory rate of 36 breaths/min in a client receiving red blood cells. An increased respiratory rate indicates a possible hemolytic transfusion reaction. The nurse must quickly stop the transfusion and assess the client further. Temperature elevations are not an indication of an allergic reaction to a platelet transfusion, although the nurse may administer acetaminophen (Tylenol) to decrease the fever. Sleepiness is expected when Benadryl is administered. Because FFP is not usually given until the PTT is 1.5 times above normal, a PTT that is 1.2 times normal indicates that the FFP has had the desired response.

The nurse is transfusing a unit of whole blood to a client when the primary health care provider prescribes "Furosemide (Lasix) 20 mg IV push." Which intervention would the nurse perform? a. Administer the furosemide after completion of the transfusion. b. Give furosemide to the client intramuscularly (IM). c. Add furosemide to the normal saline that is infusing with the blood. d. Piggyback the furosemide into the infusing blood.

a. administer the furosemide after completion of the transfusion Completing the transfusion before administering furosemide is the best course of action in this scenario. Drugs are not to be administered with infusing blood products, because they can interact with the blood, causing risks for the client. Changing the admission route is not a nursing decision. Stopping the infusing blood to administer the drug and then restarting it is also not the best decision.

A client who is receiving a blood transfusion suddenly tells the nurse, "I don't feel right!" What is the nurse's first action? a. Stop the transfusion. b. Obtain vital signs and continue to monitor. c. Call the Rapid Response Team. d. Slow the infusion rate of the transfusion.

a. stop the transfusion The nurse's first action when a client receiving a blood transfusion says, "I don't feel right," is to stop the transfusion. The client may be experiencing a transfusion reaction, so the nurse must stop the transfusion immediately. Calling the Rapid Response Team or obtaining vital signs is not the first thing that must be done. The nurse would not slow the infusion rate but would stop it altogether.

The nurse is mentoring a recent graduate registered nurse (RN) about administering blood and blood products. What action does the nurse perform before starting the transfusion? a. Verify with another RN all of the data on blood products. b. Remain with the client who is receiving the blood for the first 5 minutes of the infusion. c. Use a 22-gauge needle to obtain venous access when starting the infusion. d. Obtain the client's initial set of vital signs (VS) within the first 10 minutes of the infusion.

a. very with another RN all of the data on blood products Before administering blood and blood products, the nurse must verify with another RN all of the data on blood products. All data are checked by two RNs. Human error is the most common cause of ABO incompatibilities when administering blood and blood products. A 20-gauge needle (or a central line catheter) is used. The 22-gauge needle is too small. Initial VS must be recorded before the start of infusion of blood, not after it has begun. The nurse remains with the client for the first 15 to 30 minutes (not 5) of the infusion. This is the period when any transfusion reactions are likely to happen.

What are the risk factors for the development of leukemia? (Select all that apply.) Select all that apply. a. Multiple blood transfusions b. Bone marrow hypoplasia c. Down syndrome d. Chemical exposure e. Ionizing radiation f. Prematurity at birth

b, c, d, e, Risk factors related to the development of leukemia include: Down syndrome, chemical exposure, ionizing radiation, and bone marrow hypoplasia. Certain genetic factors contribute to the development of leukemia. Down syndrome is one such condition. Exposure to chemicals through medical need or by environmental events can also contribute. Radiation therapy for cancer or other exposure to radiation, perhaps through the environment, also contributes. Reduced production of blood cells in the bone marrow is one of the risk factors for developing leukemia. Although some genetic factors may influence the incidence of leukemia, prematurity at birth is not one of them. There is no indication that multiple blood transfusions are connected to clients who have leukemia.

The nurse is teaching a client with newly diagnosed anemia about conserving energy. Which instructions would the nurse give to the client? (Select all that apply.) Select all that apply. a. "Perform your care activities in groups to conserve your energy." b. "Allow others to perform your care during periods of extreme fatigue." c. "Drink small quantities of protein shakes and nutritional supplements daily." d. "Perform a complete bath daily to reduce your chance of getting an infection." e. "Provide yourself with four to six small, easy-to-eat meals daily." f. "Stop activity when shortness of breath or palpitations is present."

b, c, e, f Having four to six small meals daily is preferred over three large meals. This practice conserves the body's expenditure of energy used in digestion and assimilation of nutrients. Stopping activities when strain on the cardiac or respiratory system is noted is critical. It is critical to have others help the anemic client who is extremely tired. Although it may be difficult for him or her to ask for help, this practice should be stressed to the client. Drinking small protein or nutritional supplements will help rebuild the client's nutritional status. A complete bath needs to be performed only every other day. On days in between, the client can be taught to take a "mini" sponge bath, which will conserve energy and still be safe in preventing the risks for infection. Care activities would be spaced every hour or so rather than in groups to conserve energy. Care activities need to be avoided just before and after meals.

A distant family member arrives to visit a female client recently diagnosed with leukemia. The family member asks the nurse, "What should I say to her?" Which responses does the nurse suggest? (Select all that apply.) Select all that apply. a. "Tell her what you know about leukemia." b. "Talk to her as you normally would when you haven't seen her for a long time." c. "Tell her to be brave and to not cry." d. "Ask her how she is feeling." e. "Ask her if she needs anything."

b, d, e The nurse suggests some comments a family member of a recently diagnosed client with leukemia might say to the client. The first statement may be, "ask her how she is feeling." This is a broad general opening and would be nonthreatening to the client. Or "ask her if she needs anything" Asking if she needs anything is a therapeutic communication of offering self and would be considered to be therapeutic and helpful to the client. The family member would talk to her as she normally would when she hasn't seen her in a long time. There is no need to act differently with the client. If she wants to offer her feelings, keeping a normal atmosphere facilitates that option. Telling her to be brave and not to cry is callous and unfeeling. If the client is feeling vulnerable and depressed, telling her to "be brave" shuts off any opportunity for her to express her feelings. There is no need to inform the client about her disease, unless she asks about it. Opening the conversation with discussion about leukemia would be the client's prerogative

A client admitted for sickle cell crisis is distraught after learning her child also has the disease. What response by the nurse is best? a. "Both you and the father are equally responsible for passing it on." b. "I can see you are upset. I can stay here with you a while if you like." c. "It's not your fault; there is no way to know who will have this disease." d. "There are many good treatments for sickle cell disease these days."

b. "I can see you are upset. I can stay here with you a while if you like."

The nurse is teaching a client about induction therapy for acute leukemia. Which client statement indicates a need for additional education? a. "I will need to avoid people with a cold or flu." b. "After this therapy, I will not need to have any more." c. "I will probably lose my hair during this therapy." d. "The goal of this therapy is to put me in remission."

b. After this therapy I will not need to have anymore The client statement that indicates a need for additional education about induction therapy is "after this therapy, I won't need to have any more". Induction therapy is not a cure for leukemia, it is a treatment. So, the leukemia client needs more education to understand this. Because of infection risk, clients with leukemia must avoid people with a cold or flu. Induction therapy will most likely cause the client with leukemia to lose his or her hair. The goal of induction therapy is to force leukemia into remission. Because of infection risk, clients with leukemia need to avoid people with a cold or flu. Induction therapy will most likely cause the client with leukemia to lose his or her hair. The goal of induction therapy is to force leukemia into remission.

The family of a neutropenic client reports the client "is not acting right." What action by the nurse is the priority? a. Ask the client about pain. b. Assess the client for infection. c. Delegate taking a set of vital signs. d. Look at today's laboratory results.

b. Assess the client for infection.

A client admitted with a diagnosis of acute myelogenous leukemia is prescribed intravenous (IV) cytosine arabinoside for 7 days and an infusion of daunorubicin for the first 3 days. What is the major side effect of this drug therapy? a. Nausea b. Bone marrow suppression c. Liver toxicity d. Stomatitis

b. Bone marrow suppression The major side effect of this drug therapy is bone marrow suppression. Intravenous cytosine arabinoside and daunorubicin are a commonly prescribed course of aggressive chemotherapy, and bone marrow suppression is a major side effect. The client is even more at risk for infection than before treatment began. Liver toxicity, nausea, and stomatitis are not the major problems with this therapy.

A nurse in a hematology clinic is working with four clients who have polycythemia vera. Which client should the nurse see first? a. Client with a blood pressure of 180/98 mm Hg b. Client who reports shortness of breath c. Client who reports calf tenderness and swelling d. Client with a swollen and painful left great toe

b. Client who reports shortness of breath

A client with chronic anemia has had many blood transfusions. What medications does the nurse anticipate teaching the client about adding to the regimen? (Select all that apply.) a. Azacitidine (Vidaza) b. Darbepoetin alfa (Aranesp) c. Decitabine (Dacogen) d. Epoetin alfa (Epogen) e. Methylprednisolone (Solu-Medrol)

b. Darbepoetin alfa (Aranesp) d. Epoetin alfa (Epogen)

A client receiving a blood transfusion develops anxiety and low back pain. After stopping the transfusion, what action by the nurse is most important? a. Documenting the events in the client's medical record b. Double-checking the client and blood product identification c. Placing the client on strict bedrest until the pain subsides d. Reviewing the client's medical record for known allergies

b. Double-checking the client and blood product identification

A nurse is preparing to administer a blood transfusion. What action is most important? a. Correctly identifying client using two identifiers b. Ensuring informed consent is obtained if required c. Hanging the blood product with Ringer's lactate d. Staying with the client for the entire transfusion

b. Ensuring informed consent is obtained if required

A client has Hodgkin's lymphoma, Ann Arbor stage Ib. For what manifestations should the nurse assess the client? (Select all that apply.) a. Headaches b. Night sweats c. Persistent fever d. Urinary frequency e. Weight loss

b. Night sweats c. Persistent fever e. Weight loss

A client has a serum ferritin level of 8 ng/mL and microcytic red blood cells. What action by the nurse is best? a. Encourage high-protein foods. b. Perform a Hemoccult test on the client's stools. c. Offer frequent oral care. d. Prepare to administer cobalamin (vitamin B12).

b. Perform a Hemoccult test on the client's stools.

The nurse is caring for a client with neutropenia. Which clinical manifestation indicates that the client has an infection or an infection needs to be ruled out? a. Evidence of pus b. Wheezes or crackles c. Fever of 102°F (38.9°C) or higher d. Coughing and deep breathing

b. Wheezes or crackles The clinical manifestation that indicates the client with neutropenia has an infection or an infection that needs to be ruled out is wheezes or crackles. Wheezes or crackles in the neutropenic client may be the first symptom of infection in the lungs. Coughing and deep breathing are not indications of infection but can help prevent it. The client with leukopenia, not neutropenia, may have a severe infection without pus or with only a low-grade fever.

Which client does the nurse assign as a roommate for a client with aplastic anemia? a. A 23-year-old with sickle cell disease who has two draining leg ulcers b. A 28-year-old with glucose-6-phosphate dehydrogenase (G6PD) deficiency anemia who is receiving mannitol (Osmitrol) c. A 34-year-old with idiopathic thrombocytopenia who is taking steroids d. A 30-year-old with leukemia who is receiving induction chemotherapy

b. a 28 yr old with glucose-6-phosphate dehydrogenase (G6PD) deficiency anemia who is receiving mannitol (Osmitrol) The nurse assigns as a roommate to the client with aplastic anemia a 28-year-old with glucose-6-pgisphate dehydrogenase (G6PD) deficiency anemia who is receiving mannitol. Because clients with aplastic anemia usually have low white blood cell counts that place them at high risk for infection, roommates such as the client with G6PD deficiency anemia would be free from infection or infection risk. The client with sickle cell disease has two draining leg ulcer infections that would threaten the diminished immune system of the client with aplastic anemia. The client with leukemia who is receiving induction chemotherapy and the client with idiopathic thrombocytopenia who is taking steroids are at risk for development of infection, which places the client with aplastic anemia at risk, too.

A pediatric nurse is floated to a medical-surgical unit. Which client is assigned to the float nurse? a. A 55-year-old with folic acid deficiency anemia caused by alcohol abuse who needs counseling b. A 42-year-old with sickle cell disease receiving a transfusion of packed red blood cells c. A 60-year-old with newly diagnosed polycythemia vera who needs teaching about the disease d. A 50-year-old with pancytopenia needing assessment of risk factors for aplastic anemia

b. a 42 yr old with sickle cell disease receiving a transfusion of packed red blood cells The client who is assigned to the pediatric float nurse is the 42-year-old sickle cell disease client receiving a transfusion of packed blood cells. Because sickle cell disease is commonly diagnosed during childhood, the pediatric nurse will be familiar with the disease and with red blood cell transfusion. Therefore, he or she would be assigned to the client with sickle cell disease. Polycythemia vera, aplastic anemia, and folic acid deficiency are problems more commonly seen in adult clients who would be cared for by nurses who are more experienced in caring for adults.

A 56-year-oldclient admitted with a diagnosis of acute myelogenous leukemia (AML) has been prescribed intravenous (IV) cytosine arabinoside and an IV infusion of daunorubicin. The client develops an infection. Which action would the nurse take to determine that the appropriate antibiotic has been prescribed to treat this condition? a. Evaluate the client's liver function tests (LFTs) and serum creatinine levels b. Check the culture and sensitivity test results to be certain that the requested antibiotic is effective against the organism causing the infection c. Recognize that vancomycin (Vancocin) is the drug of choice used to treat all infections in clients with AML d. Monitor the client's white blood cell (WBC) count level

b. check the culture sensitivity test results to be certain that the requested antibiotic is effective against the organism causing the infection The best action the nurse takes to determine if the appropriate antibiotic has been prescribed is to check the culture and sensitivity test results to be sure that the prescribed antibiotic is effective against the organism causing the infection. Drug therapy is the main defense against infections that develop in clients undergoing therapy for AML. Agents used depend on the client's sensitivity to various antibiotics for the organism causing the infection. Although the WBC count is elevated in infection, this test does not influence which antibiotic will be effective in fighting the infection. Although LFTs and kidney function tests may be influenced by antibiotics, these tests do not determine the effectiveness of the antibiotic. Vancomycin may not be effective in all infections. Culturing of the infection site and determining the organism's sensitivity to a cohort of drugs are needed. This will provide data on drugs that are capable of eradicating the infection in this client

The nurse is caring for a client with neutropenia who has a suspected infection. Which intervention would the nurse implement first? a. Place the client on Bleeding Precautions. b. Obtain prescribed blood cultures. c. Initiate the administration of prescribed antibiotics. d. Give 1000 mL of IV normal saline to hydrate the client.

b. obtain prescribed blood cultures The intervention the nurse would first implement is to draw prescribed blood cultures. Obtaining blood cultures to identify the infectious agent correctly is the priority for this client. Placing the client on Bleeding Precautions is unnecessary. Administering antibiotics is important, but antibiotics must always be started after cultures are obtained. Hydrating the client is not the priority.

The nurse is to administer packed red blood cells to a client. How does the nurse ensure proper client identification? a. Check the client's armband b. Review all information with another registered nurse (RN) c. Ask the client's name d. Verify the client's room number

b. review all information with another registered nurse (RN) With another registered nurse, all information must be reviewed. This process includes verifying the client by name and number, checking blood compatibility, and noting the expiration time. Human error is the most common cause of ABO incompatibility reactions, even for experienced nurses. Asking the client's name and checking the client's armband are not adequate for identifying the client before transfusion therapy. Using the room number to verify client identification is never appropriate.

What are serious side effects of antiviral agents prescribed for a client with acute myelogenous leukemia? (Select all that apply.) Select all that apply. a. Cardiomyopathy b. Diarrhea c. Ototoxicity d. Nephrotoxicity e. Stroke

c, d Antiviral agents, although helpful in combating severe infection, have serious side effects, especially nephrotoxicity and ototoxicity. Cardiomyopathy and stroke are not serious side effects of antiviral agents. Diarrhea is a mild side effect associated with antibiotic therapy.

An 82-year-oldclient with anemia is prescribed 2 units of whole blood. Which assessment findings cause the nurse to discontinue the transfusion because it is unsafe for the client? (Select all that apply.) Select all that apply. a. Capillary refill less than 3 seconds b. Decreased pallor c. Rapid, bounding pulse d. Flattened superficial veins e. Hypertension f. Hypotension

c, e, f, The assessment findings that are unsafe for the nurse to continue the blood transfusion for the client are: hypotension, hypertension, and rapid, bounding pulse. In an older adult receiving a transfusion, low blood pressure is a sign of a transfusion reaction, hypertension is a sign of overload, and a rapid and bounding pulse is a sign of fluid overload. In this scenario, 2 units, or about a liter of fluid, could be problematic. Increased (not decreased) pallor and cyanosis are signs of a transfusion reaction, while swollen (not flattened) superficial veins are present in fluid overload in older adult clients receiving transfusions. Capillary refill time that is less than 3 seconds is considered to be normal and would not pose a problem.

A client has thrombocytopenia. What client statement indicates the client understands self-management of this condition? a. "I brush and use dental floss every day." b. "I chew hard candy for my dry mouth." c. "I usually put ice on bumps or bruises." d. "Nonslip socks are best when I walk."

c. "I usually put ice on bumps or bruises."

Which client is at greatest risk for experiencing a hemolytic transfusion reaction? a. A 58-year-old immune-suppressed client b. A 42-year-old client with allergies c. A 34-year-old client with type O blood d. A 78-year-old client with arthritis

c. A 34-year-old client with type O blood The client at greatest risk for experiencing a hemolytic transfusion reaction is the 34-year-old client with type O blood. Hemolytic transfusion reactions are caused by blood type or Rh incompatibility. When blood that contains antigens different from the client's own antigens is infused, antigen-antibody complexes are formed in the client's blood. Type O is considered the universal donor, but not the universal recipient. The client with allergies would be most susceptible to an allergic transfusion reaction. The older adult client with arthritis would be most susceptible to circulatory overload. The immune-suppressed client would be most susceptible to a transfusion-associated graft-versus-host disease.

A client has a platelet count of 25,000/mm3. What actions does the nurse delegate to the unlicensed assistive personnel (UAP)? (Select all that apply.) a. Assist with oral hygiene using a firm toothbrush. b. Give the client an enema if he or she is constipated. c. Help the client choose soft foods from the menu. d. Shave the male client with an electric razor. e. Use a lift sheet when needed to re-position the client.

c. Help the client choose soft foods from the menu. d. Shave the male client with an electric razor. e. Use a lift sheet when needed to re-position the client.

A nurse is caring for a client who is about to receive a bone marrow transplant. To best help the client cope with the long recovery period, what action by the nurse is best? a. Arrange a visitation schedule among friends and family. b. Explain that this process is difficult but must be endured. c. Help the client find things to hope for each day of recovery. d. Provide plenty of diversionary activities for this time.

c. Help the client find things to hope for each day of recovery.

A 32-year-old client is recovering from a sickle cell crisis. The client's discomfort is controlled with pain medications and discharge planning has been initiated. What medication will the nurse anticipate to be prescribed before discharge? a. Tissue plasminogen activator (t-PA) b. Heparin (Heparin) c. Hydroxyurea (Droxia) d. Warfarin (Coumadin)

c. Hydroxyurea (Droxia) The nurse anticipates Hydroxyurea to be prescribed for pain for a sickle cell disease client who is being discharge. Hydroxyurea (Droxia) has been used successfully to reduce sickling of cells and pain episodes associated with sickle cell disease (SCD). Clients with SCD are not prescribed anticoagulants such as heparin or warfarin (Coumadin). t-PA is used as a "clot buster" in clients who have had ischemic strokes.

The nurse is infusing platelets to a client who is scheduled for a hematopoietic stem cell transplant. What procedure does the nurse follow for administering this blood product? a. Give intravenous corticosteroids before starting the transfusion. b. Allow the platelets to stabilize at the client's bedside for 30 minutes. c. Infuse the transfusion over a 15- to 30-minute period. d. Set up the infusion with the standard transfusion Y tubing.

c. Infuse the transfusion over a 15-30 minute period The procedure the nurse follows to administer platelets to a hematopoietic stem cell transplant is to infuse the transfusion over a 15-to-30-minute period. The volume of platelets—200 or 300 mL (standard amount)—needs to be infused rapidly over a 15- to 30-minute period. A special transfusion set with a smaller filter and shorter tubing is used to get the platelets into the client quickly and efficiently. Administering steroids is not standard practice in administering platelets. Platelets must be administered immediately after they are received because they are considered to be quite fragile.

A client has a sickle cell crisis with extreme lower extremity pain. What comfort measure does the nurse delegate to the unlicensed assistive personnel (UAP)? a. Apply ice packs to the client's legs. b. Elevate the client's legs on pillows. c. Keep the lower extremities warm. d. Place elastic bandage wraps on the client's legs.

c. Keep the lower extremities warm.

A nurse is caring for a client with sickle cells disease. Which nursing action is most effective in reducing potential for sepsis in this client a. check vitals every 4 hours b. perform frequent and thorough hand washing c. administer prophylactic drug therapy d. monitor for abnormal laboratory values

c. Perform frequent and thorough hand washing The most effective nursing action to reduce the risk for sepsis in a client with sickle cell anemia is to perform frequent and thorough handwashing. Prevention and early detection strategies are used to protect the client in sickle cell crisis from infection. Frequent and thorough handwashing is of the utmost importance. Taking vital signs every 4 hours will help with early detection of infection but is not prevention. Drug therapy is a major defense against infections that develop in the client with sickle cell disease but is not the most effective way that the nurse can reduce the potential for sepsis. Continually assessing the client for infection and monitoring the daily complete blood count with differential white blood cell count is early detection, not prevention.

A client has received a bone marrow transplant and is waiting for engraftment. What actions by the nurse are most appropriate? (Select all that apply.) a. Not allowing any visitors until engraftment b. Limiting the protein in the client's diet c. Placing the client in protective precautions d. Teaching visitors appropriate hand hygiene e. Telling visitors not to bring live flowers or plants

c. Placing the client in protective precautions d. Teaching visitors appropriate hand hygiene e. Telling visitors not to bring live flowers or plants

What is the most important environmental risk for developing leukemia? a. Living near high-voltage power lines b. Direct contact with others with leukemia c. Smoking cigarettes d. Family history

c. Smoking cigarettes The most important environmental risk for developing leukemia is smoking cigarettes. According to the American Cancer Society (ACS), the only proven lifestyle-related risk factor for leukemia is cigarette smoking. Genetics is a strong indicator, but it is not an environmental risk factor. According to the ACS, living near high-voltage power lines is not a proven risk factor for leukemia. Leukemia is not contagious.

A nurse is caring for a young male client with lymphoma who is to begin treatment. What teaching topic is a priority? a. Genetic testing b. Infection prevention c. Sperm banking d. Treatment options

c. Sperm banking

A client has been treated for a deep vein thrombus and today presents to the clinic with petechiae. Laboratory results show a platelet count of 42,000/mm3. The nurse reviews the client's medication list to determine if the client is taking which drug? a. Enoxaparin (Lovenox) b. Salicylates (aspirin) c. Unfractionated heparin d. Warfarin (Coumadin)

c. Unfractionated heparin

The nurse assess the client with which hematologic condition first? a. A 40-year-old with iron deficiency anemia who needs a Z-track iron injection b. A 32-year-old with pernicious anemia who needs a vitamin B12 injection c. An 81-year-old with thrombocytopenia and an increase in abdominal girth d. A 67-year-old with acute myelocytic leukemia with petechiae on both legs

c. an 81 yr old with thrombocytopenia with an increase in abdominal girth The nurse needs to first assess the 81-year-old client with thrombocytopenia and an increase in abdominal girth. An increase in abdominal girth in a client with thrombocytopenia indicates possible hemorrhage, and warrants further assessment immediately. The 32-year-old with pernicious anemia, the 67-year-old with acute myelocytic leukemia, and the 40-year-old with iron deficiency anemia do not indicate any acute complications, so the nurse can assess them after assessing the client with thrombocytopenia.

The nurse performs an assessment on a newly admitted client with thrombocytopenia. Which assessment finding requires immediate intervention by the nurse? a. Decreased urine output b. Increased temperature c. Bleeding from the nose d. Reports of pain

c. bleeding from the nose The assessment finding on a newly admitted client with thrombocytopenia that needs immediate intervention by the nurse is bleeding from the nose. The client with thrombocytopenia has a high risk for bleeding. The nosebleed would be attended to immediately. The client's report of pain, decreased urine output, and increased temperature are not the highest priority.

The nurse is assessing the endurance level of a client in a long-term care facility. What question does the nurse ask to get this information? a. "Are your feet or hands cold, even when you are in bed?" b. "How much exercise do you get?" c. "Do you feel more tired after you get up and go to the bathroom?" d. "What is your endurance level?"

c. do you feel more tired after you get up and go to the bathroom Asking about feeling tired after using the bathroom is the best question to ask to assess a client's endurance level. This question is pertinent to the client's activity and provides a comparison. The specific activity helps the client relate to the question and provides needed answers. The hospitalized client typically does not get much exercise. This would be a difficult assessment for a client in long-term care facility to make. Asking the client about his or her endurance level is too vague. The client may not know how to answer this question. Asking about cold feet or hands does not address the client's endurance.

Which nursing intervention most effectively protects a client with thrombocytopenia a. take rectal temperatures b. avoid use of dentures c. encourage use of an electric shaver d. apply warm compresses on trauma sites

c. encourage use of electric razor The most effective nursing intervention that protects a client with thrombocytopenia is encouraging the client to use an electric shaver. This client must be advised to use an electric shaver instead of a razor. Any small cuts or nicks can cause problems because of the prolonged clotting time. To prevent rectal trauma, rectal thermometers would not be used. Oral or tympanic temperatures would be taken. Dentures may be used by clients with thrombocytopenia as long as they fit properly and do not rub. Ice (not heat) would be applied to areas of trauma.

A client who has been newly diagnosed with leukemia is admitted to the hospital. Avoiding which potential problem takes priority in the client's nursing care plan? a. Fluid overload (overhydration) b. Hemorrhage c. Infection d. Hypoxia

c. infection Avoiding infection is the priority potential problem when caring for a newly diagnosed client with leukemia. Fluid overload, hemorrhage, and hypoxia are not priority problems for the client with leukemia.

Which task is appropriate for the nurse to delegate to an unlicensed assistive personnel (UAP) working on a medical-surgical unit? a. Administer erythropoietin to a client with myelodysplastic syndrome b. Assess skin integrity on an anemic client who fell during ambulation c. Obtain vital signs on a client receiving a blood transfusion d. Assist a client with folic acid deficiency in making diet choices

c. obtain vital signs on a client receiving a blood transfusion The appropriate task for the nurse to delegate to a UAP is obtaining vital signs on a client receiving a blood transfusion. This activity is within the scope of practice for UAPs. Assisting with prescribed diet choices, administering medication, and assessing clients are complex actions that must be done by licensed nurses.

The nurse is caring for a client who is in sickle cell crisis. What action would the nurse perform first? a. Encourage the client's use of two methods of birth control. b. Apply cool compresses to the client's forehead. c. Provide pain medications as needed. d. Increase food sources of iron in the client's diet.

c. provide pain medication as needed The action the nurse would perform first for a client in sickle cell crisis is to provide pain medications as needed. Analgesics are needed to treat sickle cell pain. Cool compresses do not help the client in sickle cell crisis. Birth control is not the priority for this client. Increasing iron in the diet is not pertinent for the client in sickle cell crisis.

A client with leukemia is being discharged from the hospital. The nurse's discharge instructions say to keep regularly scheduled follow-up primary health care provider appointments. The client says, "I don't have transportation." Which is the most appropriate nursing response? a. You can take the bus b. I may be able to take you c. the local American Cancer Society may be able to help d. a pharmaceutical company might be able to help

c. the local American cancer society may be able to help The most appropriate nursing response to the client who does not have transportation for follow-up appointments is that "the local American Cancer Society may be able to help." Many local units of the American Cancer Society offer free transportation to clients with cancer, including those with leukemia. Telling the client to take the bus is dismissive and does not take into consideration the client's situation (e.g., the client may live nowhere near a bus route). Although the nurse offering to take the client is compassionate, it is not appropriate for the nurse to offer the client transportation. Suggesting a pharmaceutical company is not the best answer. Drug companies typically do not provide this type of service.

The nurse is reinforcing information about genetic counseling to a client with sickle cell disease who has a healthy spouse. What information would the nurse explain to the parents about the risk of a child having sickle cell disease? a. "Your children will have the disease, but your grandchildren will not." b. "Sickle cell disease will be inherited by your children." c. "The sickle cell trait will be inherited by your children." d. "Your children will not have the disease, but your grandchildren could."

c. the sickle cell trait will be inherited by your children The statement that explains to parents about the risk of a child having sickle cell disease is that the children of the client with sickle cell disease will inherit the sickle cell trait but may not inherit the disease. If both parents have the sickle cell trait, their children could get the disease. The children of the client with sickle cell disease will inherit the sickle cell trait but may not inherit the disease. If both parents have the sickle cell trait, their children could get the disease.

A nursing student is struggling to understand the process of graft-versus-host disease. What explanation by the nurse instructor is best? a. "Because of immunosuppression, the donor cells take over." b. "It's like a transfusion reaction because no perfect matches exist." c. "The client's cells are fighting donor cells for dominance." d. "The donor's cells are actually attacking the client's cells."

d. "The donor's cells are actually attacking the client's cells."

A nursing student is caring for a client with leukemia. The student asks why the client is still at risk for infection when the client's white blood cell count (WBC) is high. What response by the registered nurse is best? a. "If the WBCs are high, there already is an infection present." b. "The client is in a blast crisis and has too many WBCs." c. "There must be a mistake; the WBCs should be very low." d. "Those WBCs are abnormal and don't provide protection."

d. "Those WBCs are abnormal and don't provide protection."

The nurse is teaching a client with vitamin B12 deficiency anemia about dietary intake. Which type of food does the nurse encourage the client to eat? a. Grains b. Leafy vegetables c. Starchy vegetables d. Dairy products

d. Dairy products The nurse encourages the client to eat dairy products such as milk, cheese, and eggs. These foods will provide the vitamin B12 that the client needs. Grains, leafy vegetables, and starchy vegetables are not a source of vitamin B12.

The nurse is transfusing 2 units of packed red blood cells to a postoperative client. What electrolyte imbalance would the nurse monitor for after the blood transfusion? a. Hypomagnesemia b. Hyponatremia c. Hypercalcemia d. Hyperkalemia

d. Hyperkalemia The electrolyte imbalance the nurse needs to monitor after transfusing 2 units of blood to a postoperative client is hyperkalemia. During transfusion, some cells are damaged. These cells release potassium, thus raising the client's serum potassium level (hyperkalemia). This complication is especially common with packed cells and whole-blood products. High serum calcium levels, low magnesium levels, or low sodium levels are not expected with blood transfusions.

A nurse is preparing to hang a blood transfusion. Which action is most important? a. Documenting the transfusion b. Placing the client on NPO status c. Placing the client in isolation d. Putting on a pair of gloves

d. Putting on a pair of gloves

A client with autoimmune idiopathic thrombocytopenic purpura (ITP) has had a splenectomy and returned to the surgical unit 2 hours ago. The nurse assesses the client and finds the abdominal dressing saturated with blood. What action is most important? a. Preparing to administer a blood transfusion b. Reinforcing the dressing and documenting findings c. Removing the dressing and assessing the surgical site d. Taking a set of vital signs and notifying the surgeon

d. Taking a set of vital signs and notifying the surgeon

The nurse assesses a client's oral cavity and makes the discovery shown in the photo below: What action by the nurse is most appropriate? a. Encourage the client to have genetic testing. b. Instruct the client on high-fiber foods. c. Place the client in protective precautions. d. Teach the client about cobalamin therapy.

d. Teach the client about cobalamin therapy.

A client is in the preoperative holding area prior to an emergency coronary artery bypass graft (CABG). The client is yelling at family members and tells the doctor to "just get this over with" when asked to sign the consent form. What action by the nurse is best? a. Ask the family members to wait in the waiting area. b. Inform the client that this behavior is unacceptable. c. Stay out of the room to decrease the client's stress levels.

d. Tell the client that anxiety is common and that you can help. ANS: D Preoperative fear and anxiety are common prior to cardiac surgery, especially in emergent situations. The client is exhibiting anxiety, and the nurse should reassure the client that fear is common and offer to help. The other actions will not reduce the client's anxiety. DIF: Applying/Application REF: 776 KEY: Coronary artery disease| preoperative nursing| psychosocial response| anxiety| coping| therapeutic communication MSC: Integrated Process: Caring NOT: Client Needs Category: Psychosocial Integrity

A client with sickle cell disease (SCD) takes hydroxyurea (Droxia). The client presents to the clinic reporting an increase in fatigue. What laboratory result should the nurse report immediately? a. Hematocrit: 25% b. Hemoglobin: 9.2 mg/dL c. Potassium: 3.2 mEq/L d. White blood cell count: 38,000/mm3

d. White blood cell count: 38,000/mm3

A client with multiple myeloma demonstrates worsening bone density on diagnostic scans. About what drug does the nurse plan to teach this client? a. Bortezomib (Velcade) b. Dexamethasone (Decadron) c. Thalidomide (Thalomid) d. Zoledronic acid (Zometa)

d. Zoledronic acid (Zometa)

A hematology unit is staffed by registered nurse's (RNs), licensed practical nurses (LPNs)/licensed vocational nurse (LVNs), and unlicensed assistive personnel (UAP). When the nurse manager is reviewing staff documentation, which entry indicates the need for staff education for his or her appropriate level of responsibility and client care? a, "Abdominal pain relieved by morphine 4 mg IV; client resting comfortably and denies problems. B.C., RN" b. "Ambulated in hallway for 40 feet (12 m) and denies shortness of breath at rest or with ambulation. T.Y., LPN" c. "Vital signs 98.6°F (37.0°C), heart rate 60, respiratory rate 20, blood pressure 110/68, and oximetry 98% on room air. L.D., UAP" d. "Client reporting increased shortness of breath; oxygen increased to 4 L by nasal cannula. M.N., UAP"

d. client reporting increased shortness of breath; oxygen increased to 4 L by nasal cannula The documentation entry that needs education is the one from the UAP that states that the "client reports increased shortness of breath and that oxygen was increased to 4 L by nasal cannula." Determination of the need for oxygen and administration of oxygen must be done by licensed nurses who have the education and scope of practice required to administer it. All other documentation entries reflect appropriate delegation and assignment of care.

The nurse is educating a group of young women who have sickle cell disease (SCD). Which statement from a class member indicates further teaching is necessary? a. "The pneumonia vaccine is protection that I need." b. "I must take my penicillin pills as prescribed, all the time." c. "Frequent handwashing is an important habit for me to develop." d. "Getting an annual 'flu shot' would be dangerous for me."

d. getting an annual flu shot would be dangerous for me Further teaching is needed when a young women with sickle cell disease says, "Getting an annual 'flu shot' would be dangerous for me." The client with SCD can receive annual influenza and pneumonia vaccinations. This helps prevent the development of these infections, which could cause a sickle cell crisis. The pneumonia vaccine is also appropriate for the client with sickle cell disease to receive. Prophylactic penicillin is given to clients with SCD orally twice a day to prevent the development of infection. Handwashing is a very important habit for the client with SCD to develop because it reduces the risk for infection.

A recently admitted client who is in sickle cell crisis requests "something for pain." What medication would the nurse be prepared to administer? a. Oral ibuprofen (Motrin) b. Intramuscular (IM) morphine sulfate c. Oral morphine sulfate (MS-Contin) d. Intravenous (IV) hydromorphone (Dilaudid)

d. intravenous (IV) hydromorphone (dilaudid) The client with sickle cell disease needs IV pain relief, and it needs to be administered on a routine schedule (i.e., before the client has to request it). Nonsteroidal anti-inflammatory drugs may be used for clients with SCD for pain relief once their pain is under control. However, in a crisis, this choice of analgesic is not strong enough. Moderate pain may be treated with oral opioids, but this client is in a sickle cell crisis. IV analgesics would be used until his or her condition stabilizes. Morphine is not administered intramuscularly (IM) to clients with sickle cell disease (SCD). In fact, all IM injections are avoided because absorption is impaired by poor perfusion and sclerosed skin.

A client with thrombocytopenia is being discharged. Which instruction would the nurse include in a teaching plan for this client? a. "Drink at least 2 L of fluid per day." b. "Avoid large crowds." c. "Elevate your lower extremities when sitting." d. "Use a soft-bristled toothbrush."

d. use a soft bristled toothbrush Using a soft-bristled toothbrush reduces the risk for bleeding in the client with thrombocytopenia. Avoiding large crowds reduces the risk for infection but is not specific to the client with thrombocytopenia. Increased fluid intake reduces the risk for dehydration but is not particularly relevant to the client with thrombocytopenia. Elevating extremities reduces the risk for dependent edema but is not specific to the client with thrombocytopenia.

A client with multiple myeloma reports bone pain that is unrelieved by analgesics. What is the most appropriate response by the nurse? a. "Ask your doctor to prescribe more medication." b. "It is too soon for additional medication to be given." c. "I'll turn on some soothing classical music for you." d. "Would you like to try some relaxation techniques?"

d. would you like to try some relaxation techniques he most appropriate response by the nurse to the client with multiple myeloma is "would you like to try some relaxation techniques"? Because most clients with multiple myeloma have local or generalized bone pain, analgesics, and alternative approaches for pain management, such as relaxation techniques are used for pain relief. This also offers the client a choice. Before prescribing additional medication, other avenues would be explored to relieve this client's pain. Although music therapy can be helpful, this response does not give the client a choice. Even if it is too soon to give additional medication, telling that to the client is not helpful because it dismisses the client's pain concerns.

A nurse prepares a client for a pharmacologic stress echocardiogram. Which actions should the nurse take when preparing this client for the procedure? (Select all that apply.) a. Assist the provider to place a central venous access device. b. Prepare for continuous blood pressure and pulse monitoring. c. Administer the client's prescribed beta blocker. d. Give the client nothing by mouth 3 to 6 hours before the procedure.

e. Explain to the client that dobutamine will simulate exercise for this examination. ANS: B, D, E Clients receiving a pharmacologic stress echocardiogram will need peripheral venous access and continuous blood pressure and pulse monitoring. The client must be NPO 3 to 6 hours prior to the procedure. Education about dobutamine, which will be administered during the procedure, should be performed. Beta blockers are often held prior to the procedure. DIF: Applying/Application REF: 646 KEY: Assessment/diagnostic examination| medication MSC: Integrated Process: Nursing Process: Planning NOT: Client Needs Category: Physiological Integrity: Pharmacological and Parenteral Therapies


Kaugnay na mga set ng pag-aaral

All Nutrition Quizzes for Test #1

View Set

Wiley Plus Chapter 25 Animations & Test Bank

View Set

Due Process and Right to Privacy

View Set

Community Chapter 11 NCLEX Questions

View Set

Complete Commercial Law (2nd Half)

View Set

Trigonometric Ratios (0, 30, 45, 60, 90, 180, 270)

View Set

Chapter 21 the rise of progressivism

View Set